Крок 3 - Медицина 2016 весна (буклет)

1 / 200
До приймального відділення військового госпіталю доставлено військовослужбовця з вогнепальним пораненням в ділянку черевної порожнини. Шкіра бліда, АТ- 90/60 мм рт.ст., Ps-100/хв. Симптоми подразнення очеревини сумнівні. Яке дослідження треба виконати для уточнення діагнозу? A serviceman with a gunshot wound in the abdominal cavity was brought to the reception department of the military hospital. The skin is pale, blood pressure - 90/60 mm Hg, Ps-100/ min. Symptoms of peritoneal irritation are doubtful. What research should be performed to clarify the diagnosis?

Комп’ютерна томографія органів черевної порожнини Computer tomography of abdominal organs

Фіброгастродуоденоскопія Fibrogastroduodenoscopy

УЗД органів черевної порожнини Ultrasound of abdominal organs

Оглядова рентгеноскопія органів черевної порожнини Rentoscopy of abdominal organs

Лапаротомія Laparotomy

2 / 200
До медичної роти доставлено постраждалого з тяжким вогнепальним пораненням нижньої третини правого стегна. Кістка роздроблена. Магістральні судини пошкоджені. Кінцівка висить на клапті м’яких тканин. До якого виду медичної допомоги відноситься проведення відсікання кінцівки, яка висить на клапті м’яких тканин? A victim with a severe gunshot wound to the lower third of the right thigh was brought to the medical unit. The bone is crushed. The trunk vessels are damaged. The limb hangs on a flap of soft tissue. What species 'Does medical care include amputation of a limb that hangs on a flap of soft tissue?

Кваліфікована медична допомога Qualified medical assistance

Спеціалізована медична допомога Specialized medical care

Перша медична допомога First aid

Перша лікарська допомога First medical aid

Долікарська (фельдшерська) медична допомога Paramedic (paramedic) medical assistance

3 / 200
У пацієнта з симптомами відсутності кровообігу на ЕКГ встановлена асистолія. Першочерговим терапевтичним заходом має бути: A patient with symptoms of lack of blood circulation on the ECG has asystole. The first therapeutic measure should be:

Електрична дефібриляція Electrical defibrillation

Введення хлориду кальцію Introduction of calcium chloride

Введення лідокаїну Lidocaine administration

Введення адреналіну Injection of adrenaline

Введення атропіну Atropine administration

4 / 200
Травмований мотоцикліст лежить на проїжджій частині дороги. Ліве стегно його деформоване в нижній третині, через зовнішню рану фонтанує кров. Де і який кровоспинний джгут необхідно застосувати? The injured motorcyclist is lying on the road. His left thigh is deformed in the lower third, blood is pouring out of the external wound. Where and what kind of tourniquet should be applied?

Венозний джгут на верхню третину гомілки Venous tourniquet on the upper third of the lower leg

Артеріальний джгут в середній третині стегна Arterial tourniquet in the middle third of the thigh

Артеріальний джгут безпосередньо над раною стегна Arterial tourniquet directly above thigh wound

Венозний джгут безпосередньо над раною Venous tourniquet directly above the wound

Артеріальний джгут в верхній третині стегна Arterial tourniquet in the upper third of the thigh

5 / 200
У хворого, який перебуває в гіперглікемічній комі, через 4 години після початку інсулінотерапії раптово розвинулися тахікардія, м’язова гіпотонія; АТ- 80/40 мм рт.ст. На ЕКГ: подовження інтервалу QT, зниження S — T, розширення і сплощення T. Який найбільш імовірний діагноз? In a patient who is in a hyperglycemic coma, 4 hours after the start of insulin therapy, tachycardia and muscle hypotension suddenly developed; blood pressure - 80/40 mm Hg On the ECG: prolongation of the QT interval, reduction of S — T, expansion and flattening of T. What is the most likely diagnosis?

Гіперглікемія Hyperglycemia

Гіповолемія Hypovolemia

Гіпокаліємія Hypokalemia

Гіперкаліємія Hyperkalemia

Гіпоглікемія Hypoglycemia

6 / 200
Вночі у хворого з’явилися задуха, кашель з рясним харкотинням рожевого кольору. З анамнезу: страждає на гіпертонічну хворобу, варикозне розширення вен обох кінцівок, 2 роки тому переніс інфаркт міокарда. Об’єктивно: неспокійний, положення ортопное, акроціаноз, ЧДР- 40/хв., АТ- 220/110 мм рт.ст. У легенях - різнокаліберні середньо- і великопухирчасті хрипи. Який найбільш імовірний діагноз? At night, the patient developed shortness of breath, a cough with copious sputum of pink color. From the anamnesis: he suffers from hypertension, varicose veins of both extremities, 2 years ago he suffered myocardial infarction. Objectively: restless, orthopneic position, acrocyanosis, systolic blood pressure - 40/min., BP - 220/110 mm Hg. In the lungs - medium- and large-scale rales of various calibers. What is the most likely diagnosis?

Астматичний статус Asthmatic status

Гостра лівошлуночкова недостатність Acute left ventricular failure

Тромбоемболія легеневої артерії Thromboembolism of the pulmonary artery

Крупозна пневмонія Croup pneumonia

Легенева кровотеча Pulmonary hemorrhage

7 / 200
До приймального відділення лікарні звернувся постраждалий у стані легкого алкогольного сп’яніння зі скаргами на порушення зору, що виникли після вживання спиртного. Найбільш імовірний діагноз: A victim in a state of mild alcohol intoxication came to the reception department of the hospital with complaints of visual disturbances that occurred after drinking alcohol. The most likely diagnosis:

Гострий напад глаукоми Acute attack of glaucoma

Отруєння етиленгліколем Ethylene glycol poisoning

Отруєння етанолом Ethanol poisoning

Гостре порушення мозкового кровообігу Acute cerebrovascular accident

Отруєння метанолом Methanol poisoning

8 / 200
У пацієнта 40-ка років, який впродовж 17-ти років хворіє на неспецифічний виразковий коліт, при останньому ендоскопічному дослідженні виявлено множинні запальні псевдополіпи у всіх відділах товстої кишки. Результати біопсії показали наявність в поліпах дисплазії II-III ст. Яке лікування необхідно застосувати в даного хворого? In a 40-year-old patient who has been suffering from non-specific ulcerative colitis for 17 years, the last endoscopic examination revealed multiple inflammatory pseudopolyps in all sections of the colon. The results of the biopsy showed the presence of stage II-III dysplasia in the polyps. What treatment should be applied to this patient?

Хірургічне Surgical

Медикаментозне Medical

Хіміотерапія Chemotherapy

Рентгенотерапія X-ray therapy

Трансендоскопічне видалення поліпів Transendoscopic removal of polyps

9 / 200
Постраждалий 42-х років, виявлений на місці ДТП, скаржиться на біль у ділянці таза та правого стегна. Об’єктивно: загальмований, стогне, шкіра бліда, язик сухий. ЧДР- 28/хв. Праве стегно деформоване, набрякле, у верхній його третині патологічна рухливість, крепітація кісткових уламків. Великий крововилив, набряк та біль під час пальпації в правих пахвинній, здухвинній та сідничній ділянках. Ps-112/хв., АТ- 80/45 мм рт.ст. Якому ступеню травматичного шоку відповідає стан потерпілого? A 42-year-old victim found at the scene of an accident complains of pain in the pelvis and right thigh. Objectively: slowed down, moaning, pale skin, tongue dry. ChDR- 28/min. The right thigh is deformed, swollen, in its upper third there is pathological mobility, crepitation of bone fragments. Large hemorrhage, swelling and pain during palpation in the right inguinal, pubic and buttock areas. Ps-112/min. Blood pressure - 80/45 mm Hg. To what degree of traumatic shock does the victim's condition correspond?

III III

- -

II II

IV IV

I I

10 / 200
Дитина 6-ти років госпіталізована до дитячого відділення з бронхопневмонією. Страждає на атопічний дерматит. Після внутрішньом’язового введення ампіциліну з’явилися відчуття стиснення у грудях, запаморочення, різка блідість, ціаноз, холодний піт, прискорене шумне дихання. Який з перерахованих препаратів слід ввести в маніпуляційній негайно? A 6-year-old child was hospitalized in the children's ward with bronchopneumonia. He suffers from atopic dermatitis. After intramuscular administration of ampicillin, chest tightness, dizziness, sharp pallor, cyanosis, cold sweat, accelerated noisy breathing. Which of the listed drugs should be administered in the manipulation room immediately?

Розчин тавегілу внутрішньовенно Tavegil intravenous solution

Розчин адреналіну підшкірно Solution of adrenaline subcutaneously

Розчин еуфіліну внутрішньовенно Euphilin intravenous solution

Розчин преднізолону внутрішньовенно Prednisone intravenous solution

Розчин строфантину внутрішньовенно Strophanthine intravenous solution

11 / 200
У дитини 2-х років у стаціонарі на фоні гострої респіраторної вірусної інфекції з явищами риніту, фарингіту, трахеїту, температури тіла - 39,6oC, розвинувся приступ генералізованих судом. Який метод невідкладних протисудомних заходів слід вважати методом вибору? A 2-year-old child in a hospital against the background of an acute respiratory viral infection with rhinitis, pharyngitis, tracheitis, body temperature - 39.6oC, developed an attack of generalized convulsions What method of emergency anticonvulsant measures should be considered the method of choice?

Дроперидол довенно Droperidol for a long time

Фенобарбітал зондом у шлунок Phenobarbital by gastric tube

Діазепам довенно Diazepam daily

Діазепам дом’язово Intramuscular Diazepam

Хлоралгідрат у клізмі Chloral hydrate enema

12 / 200
У хворого болі в грудній клітці, які виникли після травми, задишка, слабкість. Об’єктивно: тахіпное, відставання правої половини грудної клітки в диханні, розширені вени шиї. Прогресує дихальна недостатність. Рентгенологічно: відсутність легеневого малюнка правої легені, середостіння зміщене вліво, на вдиху зміщується вправо. Купол діафрагми зміщений донизу. Який найбільш імовірний діагноз? The patient has chest pains that occurred after an injury, shortness of breath, weakness. Objectively: tachypnea, lagging of the right half of the chest in breathing, enlarged neck veins . Respiratory insufficiency is progressing. X-ray: absence of a pulmonary pattern of the right lung, the mediastinum is shifted to the left, on inhalation it shifts to the right. The dome of the diaphragm is shifted downward. What is the most likely diagnosis?

Закритий пневмоторакс Closed pneumothorax

Напружений пневмоторакс Tense pneumothorax

Емболія легеневої артерії Pulmonary embolism

Гемопневмоторакс Hemopneumothorax

Емфізема середостіння Emphysema of the mediastinum

13 / 200
Хворий 56-ти років госпіталізований зі скаргами на відчуття важкості в верхніх відділах живота, схуднення, періодичні блювання з’їденою їжею. Виразковий анамнез - 30 років. При пальпації визначається ”шум плескоту” натщесерце. Добовий діурез до 800 мл на добу. При рентгендослідженні шлунка - затримання барієвої суспензії в шлунку до 14 годин, деформація пілородуоденальної зони. Який діагноз найбільш імовірний? A 56-year-old patient was hospitalized with complaints of heaviness in the upper abdomen, weight loss, periodic vomiting of eaten food. Ulcer history - 30 years. On palpation a 'cluttering noise' is determined on an empty stomach. Daily diuresis up to 800 ml per day. X-ray examination of the stomach - retention of barium suspension in the stomach for up to 14 hours, deformation of the pyloroduodenal zone. What is the most likely diagnosis?

Гострий пієлонефрит Acute pyelonephritis

Субкомпенсований виразковий пілоростеноз Subcompensated ulcerative pyloric stenosis

Гострий панкреатит Acute pancreatitis

Компенсований виразковий пілоростеноз Compensated ulcerative pyloric stenosis

Декомпенсований виразковий пілоростеноз Decompensated ulcerative pyloric stenosis

14 / 200
У хворого з інфекційним ендокардитом з ураженням аортального клапану виник інтенсивний біль у попереку, зліва та справа, який був розцінений як інфаркт нирок. Протягом доби у хворого посилились загальна слабкість, апатія, тахікардія, артеріальний тиск знизився до 60/0 мм рт.ст., з’явилися блювання, пронос, холодний липкий піт, олігурія. Чим ускладнився перебіг захворювання? A patient with infectious endocarditis with damage to the aortic valve developed intense pain in the lower back, left and right, which was regarded as a kidney infarction. Over the course of a day, the patient's general weakness worsened , apathy, tachycardia, blood pressure decreased to 60/0 mm Hg, vomiting, diarrhea, cold sticky sweat, oliguria appeared. What complicated the course of the disease?

Гостра ниркова недостатність внаслідок інфаркту нирки Acute renal failure due to renal infarction

Гостра серцева недостатність Acute heart failure

Гостра надниркова недостатність внаслідок крововиливу у наднирники Acute adrenal insufficiency due to adrenal hemorrhage

Інфаркт мозку Cerebral infarction

Інфаркт селезінки Splenic infarction

15 / 200
До лікарні бригадою ШМД доставлено жінку 32-х років. З анамнезу: протягом останніх 2х років перебувала на диспансерному обліку з приводу аутоімунної гемолітичної анемії. Об’єктивно: стан середньої важкості, температура - 37,3oC, шкіра лимонно-жовтого кольору, Ps- 98/хв., АТ- 80/55 мм рт.ст., збільшена селезінка. У крові: ер.- 2,6•1012/л, Hb- 70 г/л, КП- 0,94, лейк.- 6,1•109/л, ШОЕ- 19 мм/год, непрямий білірубін - 58 мкмоль/л. Що необхідно призначити в першу чергу? A 32-year-old woman was brought to the hospital by the medical team. From the anamnesis: for the past 2 years, she has been under medical records for autoimmune hemolytic anemia. Objectively: the condition of moderate severity, temperature - 37.3oC, lemon-yellow skin, Ps - 98/min., BP - 80/55 mm Hg, enlarged spleen. In blood: er. - 2.6•1012/l, Hb - 70 g/l, KP - 0.94, leuk. - 6.1•109/l, ESR - 19 mm/h, indirect bilirubin - 58 μmol/l. What should be prescribed first of all?

Преднізолон Prednisone

Інтерферон Interferon

Ферум-лек Ferum-lek

Еритроцитарна маса Erythrocyte mass

Трансфузія консервованої крові Transfusion of canned blood

16 / 200
Чоловік 36-ти років скаржиться на інтенсивний біль у епігастральній ділянці, багаторазове блювання, яке не приносить полегшення, виражену загальну слабкість. Об’єктивно: загальний стан важкий, шкіра та видимі слизові - бліді, з поодинокими фіолетовими плямами. Ps- 100/хв., АТ- 100/60 мм рт.ст. Під час пальпації живота визначається ригідність черевної стінки та різка болючість у епігастральній ділянці на 6 см вище пупка та у лівому реберно-хребетному куті. Пульсація аорти у епігастральній ділянці не визначається. Який найбільш імовірний діагноз? A 36-year-old man complains of intense pain in the epigastric area, repeated vomiting that does not bring relief, marked general weakness. Objectively: the general condition is severe, skin and visible mucous membranes - pale, with single purple spots. Ps- 100/min., BP- 100/60 mm Hg. During palpation of the abdomen, rigidity of the abdominal wall and sharp pain in the epigastric area 6 cm above the navel and in in the left costo-spinal angle. Pulsation of the aorta in the epigastric area is not determined. What is the most likely diagnosis?

Розшаровуюча аневризма аорти Dissecting aortic aneurysm

Гострий холецистит Acute cholecystitis

Перфоративна виразка Perforating ulcer

Гострий панкреатит Acute pancreatitis

Гострий апендицит Acute appendicitis

17 / 200
Хворий 50-ти років після аварії на виробництві був госпіталізований із скаргами на металевий присмак та печіння у роті, нудоту, слинотечу, багаторазове блювання, пронос з домішками слизу та крові, біль у епігастральній та поперековій ділянках. Об’єктивно: стан важкий, виразковий стоматит, гінгівіт, некроз слизової оболонки носа, набряк гортані. АТ- 150/100 мм рт.ст., Ps- 48/хв., тони серця глухі. Поліурія, гіпостенурія, альбумінурія, гематурія. Який антидот слід ввести хворому? A 50-year-old patient was hospitalized after an industrial accident with complaints of a metallic taste and burning in the mouth, nausea, drooling, repeated vomiting, diarrhea with mucus impurities and of blood, pain in the epigastric and lumbar regions. Objectively: the condition is severe, ulcerative stomatitis, gingivitis, necrosis of the nasal mucosa, swelling of the larynx. BP - 150/100 mm Hg, Ps - 48/min, heart sounds are dull Polyuria, hyposthenuria, albuminuria, hematuria. What antidote should be administered to the patient?

Атропіну сульфат Atropine sulfate

Дієтиоксим Dietioxime

Хромосмон Chromosmon

Унітіол Unithiol

Антарсин Antarsyn

18 / 200
Хворому 52-х років виконана екскреторна урографія. Через 10 хвилин після внутрішньовенного введення контрастної речовини стан хворого різко погіршився. Непритомний. АТ-50/20 мм рт.ст., Ps- 120/хв., ритмічний. В анамнезі - медикаментозна алергія. З метою лікування необхідно ввести: A 52-year-old patient underwent excretory urography. 10 minutes after the intravenous injection of a contrast agent, the patient's condition worsened sharply. He is unconscious. BP-50/20 mm Hg ., Ps- 120/min., rhythmic. In the anamnesis - drug allergy. For the purpose of treatment, it is necessary to enter:

Внутрішньовенно преднізолон, інфузія кристалоїдів Intravenous prednisone, crystalloid infusion

Адреналін підшкірно, димедрол внутрішньовенно Adrenaline subcutaneously, diphenhydramine intravenously

Адреналін внутрішньовенно, швидка інфузія кристалоїдів Adrenaline IV, rapid infusion of crystalloids

Внутрішньом’язово димедрол, преднізолон Intramuscular diphenhydramine, prednisolone

Внутрішньовенно еуфілін Intravenous euphilin

19 / 200
Хворий 48-ми років скаржиться на задишку, що раптово посилилася, слабкість. Знаходиться у кардіологічному відділенні впродовж 12-ти годин, отримує: гепарин, аспірин, стрептокіназу. Збуджений, шкіра холодна, волога, ціаноз. Пульс слабкого наповнення, ритмічний, 135/хв., АТ- 60/40 мм рт.ст. У нижніх відділах легень вологі хрипи. На ЕКГ у відведеннях V3 - V6 комплекс QRS типу QS, підйом сегменту ST вище ізолінії, позитивний зубець T. З якого препарату необхідно починати невідкладну терапію? A 48-year-old patient complains of shortness of breath, which suddenly worsened, weakness. He is in the cardiology department for 12 hours, receives: heparin, aspirin, streptokinase. Excited, the skin is cold, moist, cyanosis. The pulse is weakly filled, rhythmic, 135/min., BP- 60/40 mm Hg. In the lower parts of the lungs, moist rales. On the ECG, in leads V3 - V6, the QRS complex of the QS type, rise the ST segment is above the isoline, the T wave is positive. With which drug should emergency therapy be started?

Верапаміл Verapamil

Допамін Dopamine

Дигоксин Digoxin

Лазикс Lasix

Альбумін Albumin

20 / 200
До лікаря звернувся хворий 62-х років, який після тривалого перебування в темряві та емоційного напруження скаржиться на сильний біль в лівому оці, в лівій половині голови, затуманювання зору, нудоту, блювання. Різко знижена гострота зору лівого ока, на очному яблуці змішана ін’єкція, рогова оболонка набрякла, передня камера мілка, зіниця розширена, внутрішньоочний тиск пальпаторно різко підвищений. Для якої патології характерна така клінічна картина? A 62-year-old patient came to the doctor, who after a long stay in the dark and emotional stress complains of severe pain in the left eye, in the left half of the head, blurred vision , nausea, vomiting. The visual acuity of the left eye is sharply reduced, there is a mixed injection on the eyeball, the cornea is swollen, the anterior chamber is shallow, the pupil is dilated, the intraocular pressure on palpation is sharply increased. For what pathology is this clinical picture characteristic?

Гострий кон’юнктивіт Acute conjunctivitis

Іридоцикліт з гіпертензією Iridocyclitis with hypertension

Гострий напад глаукоми Acute attack of glaucoma

Іридоциклгг Iridocyclegg

Кератит Keratitis

21 / 200
Чоловіка 40-ка років доставлено до приймального відділення. Відзначається запах алкоголю з рота, відкривання очей та мовні відповіді відсутні, нецілеспрямований рух на біль. Об’єктивно: дихання часте (>40/хв.), поверхневе, АТ- 90/40 мм рт.ст., Ps- 112/хв. Що треба зробити в першу чергу? A 40-year-old man was brought to the reception department. The smell of alcohol from the mouth is noted, there is no opening of the eyes and speech responses, no purposeful movement due to pain. Objectively: breathing frequent (>40/min.), superficial, BP - 90/40 mm Hg, Ps - 112/min. What should be done first of all?

Зондове промивання шлунку Gastric tube lavage

Форсований діурез Forced diuresis

Позаниркові методи детоксикації (гемодіаліз, гемосорбція) Extrarenal detoxification methods (hemodialysis, hemosorption)

Внутрішньовенно бемегрід Intravenous bemegrid

Інтубація трахеї Tracheal intubation

22 / 200
Бригада швидкої допомоги була викликана до 8-місячної дитини у зв’язку з тим, що мама звернулась зі скаргами на періодичне посмикування підборіддя у дитини, яке посилюється під час неспокою, посмикування окремих груп м’язів, здригання під час сну. При об’єктивному огляді виявлені ознаки рахіту II ступеня. Анамнестично з’ясовано, що дитина знаходилася на молочно-вуглеводному вигодовуванні, профілактику рахіту не проводили. Діагностовано спазмофілію. Вкажіть препарат вибору для зняття судомного синдрому: The emergency team was called to an 8-month-old child due to the fact that the mother complained of periodic twitching of the child's chin, which worsens during restlessness, twitching of certain muscle groups, tremors during sleep. An objective examination revealed signs of II degree rickets. Anamnesis revealed that the child was on milk-carbohydrate feeding, rickets prevention was not carried out. Spasmophilia was diagnosed. Specify the drug of choice to relieve convulsive syndrome:

Фенобарбітал Phenobarbital

Седуксен Seduxen

Натрію оксибутірат Sodium oxybutyrate

Кальцію глюконат Calcium gluconate

Сульфат магнію Magnesium sulfate

23 / 200
Вагітна 17-ти років у терміні 34-35 тижнів доставлена бригадою ШМД в пологовий будинок зі скаргами на головний біль, погіршення зору, посмикування м’язів, судоми. АТ190/100 мм рт.ст. на обох руках, набряки на ногах та животі. Стан плоду не порушений, виділень із статевих органів немає. Який найбільш імовірний діагноз? A 17-year-old pregnant woman at 34-35 weeks of age was taken to the maternity hospital by the medical team with complaints of headache, vision impairment, muscle twitching, convulsions. Blood pressure 190/100 mm Hg on both arms, swelling on the legs and abdomen. The condition of the fetus is not disturbed, there are no discharges from the genitals. What is the most likely diagnosis?

Прееклампсія тяжкого ступеня Severe preeclampsia

Гіпертонічна енцефалопатія Hypertensive encephalopathy

Еклампсія Eclampsia

Менінгоенцефаліт Meningoencephalitis

Епілепсія Epilepsy

24 / 200
Жінка з терміном вагітності 6 тижнів скаржиться на спрагу, свербіння шкіри, порушення зору, поліурію. В анамнезі I тип цукрового діабету. В аналізі крові: цукор 9 ммоль/л натще, після навантаження глюкозою -12 ммоль/л. Огляд окуліста: проліферативна ретинопатія. Що необхідно запропонувати жінці? A 6-week pregnant woman complains of thirst, itchy skin, visual disturbances, polyuria. She has a history of type I diabetes. In the blood test: sugar 9 mmol/ l on an empty stomach, after a glucose load -12 mmol/l. Ophthalmologist's examination: proliferative retinopathy. What should be offered to a woman?

Перервати вагітність Abort pregnancy

Лікування дієтою Diet treatment

Зберігання вагітності Pregnancy Storage

Подальше обстеження Further examination

Лікування інсуліном Insulin treatment

25 / 200
Пологи другі, своєчасні. 3 години тому відійшли навколоплідні води. Пологова діяльність активна, загальнорівномірнозвужений таз II ступеня. Передбачувана маса плоду 3900 г. Серцебиття плоду не вислуховується. При піхвовому дослідженні відкриття шийки матки повне, плідного міхура немає, передлежить голівка плоду, притиснута до входу в малий таз. Стрілоподібний шов у поперечному розмірі. Яка лікарська тактика? Second birth, timely. 3 hours ago, the amniotic fluid left. Labor activity is active, generally uniformly narrowed pelvis of the II degree. Estimated weight of the fetus is 3900 g. The heartbeat of the fetus is not heard. When vaginal examination, the opening of the cervix is ​​complete, there is no amniotic sac, the head of the fetus is presented, pressed against the entrance to the small pelvis. An arrow-shaped seam in the transverse dimension. What are the medical tactics?

Накладання акушерських щипців Applying obstetric forceps

Ведення пологів через природні статеві шляхи Giving birth through the natural genital tract

Краніотомія Craniotomy

Стимуляція пологової діяльності окситоцином Stimulation of labor with oxytocin

Кесарів розтин Caesarean section

26 / 200
На другу добу після субтотальної резекції щитоподібної залози з приводу дифузного токсичного зобу у хворої з’явилися тонічні судоми пальців (”рука акушера”), позитивні симптоми Хвостека, Вейса, Труссо. Яке ускладнення операції виникло? On the second day after subtotal resection of the thyroid gland due to diffuse toxic goiter, the patient developed tonic spasms of the fingers ('hand of the obstetrician'), positive symptoms of Khvostek, Weiss , Trousseau. What complication of the operation occurred?

Асфіксія Asphyxia

Гіпопаратиреоз Hypoparathyroidism

Анафілактичний шок Anaphylactic shock

Інфаркт міокарда Myocardial infarction

Тиреотоксичний криз Thyrotoxic crisis

27 / 200
У хворого на епілепсію в стаціонарі виник статус генералізованих тоніко-клонічних припадків. Який з немедикаментозних заходів може бути використаний для покращення стану хворого? The status of generalized tonic-clonic seizures has developed in an epileptic patient in a hospital. Which of the non-pharmacological measures can be used to improve the patient's condition?

Магнітотерапія Magnetic therapy

Бальнеотерапія Balneotherapy

Краніо-церебральна гіпотермія Cranio-cerebral hypothermia

Су-джок терапiя Su-jock therapy

Електрофорез комірцевої зони Electrophoresis of the collar zone

28 / 200
До лікаря швидкої допомоги звернулася роділля після домашніх пологів. Зі слів відомо, що за 30 хвилин тому в неї народилася дівчинка, після чого через 10 хвилин виділився послід. Почалася кровотеча. Які дії лікаря? A woman in labor went to the emergency room doctor after a home birth. It is known from the words that she gave birth to a girl 30 minutes ago, after which 10 minutes later the litter came out. It started bleeding. What are the doctor's actions?

Ввести метилергометрін і госпіталізувати роділлю Enter methylergometrine and hospitalize for childbirth

Ввести ергометрін та спостерігати Enter ergometrine and observe

Ввести ензапрост та спостерігати Enter enzaprost and observe

Ввести окситоцин та спостерігати Inject oxytocin and observe

Госпіталізувати роділлю Hospitalize for childbirth

29 / 200
У дитини 9-ти років, яка хворіє на цукровий діабет, на тлі кишкової інфекції виникли і наростають неврологічні симптоми: порушення орієнтації, галюцинації, фокальні судоми. Наявні ознаки дегідратації III ступеня, дихальних порушень немає, запах ацетону відсутній. Який попередній діагноз? A 9-year-old child with diabetes developed and is increasing neurological symptoms against the background of an intestinal infection: disorientation, hallucinations, focal convulsions. Present signs dehydration of the III degree, there are no respiratory disorders, there is no smell of acetone. What is the previous diagnosis?

Гіперосмолярна кома Hyperosmolar coma

Пухлина головного мозку Brain tumor

Гіпоглікемічна кома Hypoglycemic coma

Енцефаліт Encephalitis

Діабетична кетоацидотична кома Diabetic ketoacidotic coma

30 / 200
У хлопчика 13-ти років, що страждає на виразкову хворобу дванадцятипалої кишки, вдома виникла клініка шлунково-кишкової кровотечі. Яка подальша тактика? A 13-year-old boy suffering from duodenal ulcer has developed gastrointestinal bleeding at home. What are the next tactics?

Госпіталізація дитини у реанімаційне відділення Hospitalization of a child in the intensive care unit

Введення серцевих глікозидів Introduction of cardiac glycosides

Спостереження за дитиною Child Monitoring

Введення глюкокортикоїдів Introduction of glucocorticoids

Проведення гемостатичної терапії Hemostatic therapy

31 / 200
У дитини 8-ми років підвищення температури до 39oC, нежить зі значними гнійними виділеннями, вологий кашель, кон’юнктивіт з гнійним виділенням, світлобоязливість. На слизовій оболонці щік білуваті ділянки висівкоподібно злущеного епітелію. Який найбільш імовірний діагноз? An 8-year-old child has a temperature rise to 39oC, runny nose with significant purulent discharge, wet cough, conjunctivitis with purulent discharge, photophobia. On the mucous membrane of the cheeks whitish areas of squamous desquamated epithelium. What is the most likely diagnosis?

Грип Flu

Кореподібна краснуха Crush rubella

Скарлатина Scarlatina

Кір Measles

Висипний тиф Typhoid

32 / 200
У дитини 4-тижневого віку через 2 тижні від початку захворювання, що проявляється блюванням 'фонтаном' вираженою гіпотрофією і ексикозом, педіатр припустив вроджений пілоростеноз. При надходженні до дитячого відділення стан вкрай важкий: риси обличчя загострені, адинамія, сильна спрага, анурія, запах сечі з рота, сірість шкірних покривів. У сироватці крові: натрій -135 моль/л, калій - 3,7 ммоль/л. Коматозний стан якого характеру розвинувся у дитини? In a 4-week-old child, 2 weeks after the onset of the disease, manifested by 'fountain' vomiting with pronounced hypotrophy and exicosis, the pediatrician suspected congenital pylorostenosis. Upon admission to the children's department, the condition is extremely severe: facial features are sharpened, adynamia, strong thirst, anuria, the smell of urine from the mouth, gray skin. In the blood serum: sodium -135 mol/l, potassium - 3.7 mmol/l. What kind of comatose state has developed does the child have?

Азотемічна кома Azotemic coma

Гіперосмолярна кома Hyperosmolar coma

Кетоацидотична кома Ketoacidotic coma

Гіпоглікемічна кома Hypoglycemic coma

Гіпохлоремічна кома Hypochloremic coma

33 / 200
Військовослужбовець 36-ти років доставлений в хірургічне відділення з вогнепальним пораненням правого стегна і масивною кровотечею із рани. В операційній під час ревізії встановлено наявність повного дефекту стегнової вени на протязі 3 см. Який метод тимчасової зупинки кровотечі є найбільш ефективний? A 36-year-old military serviceman was taken to the surgical department with a gunshot wound to the right thigh and massive bleeding from the wound. In the operating room, during the audit, the presence of a complete defect of the femoral vein along the 3 cm. Which method of temporarily stopping bleeding is the most effective?

Накладання турнікетів на кінці вени Applying tourniquets at the end of the vein

Накладання затискача на проксимальний відрізок вени Applying the clamp on the proximal segment of the vein

Тимчасове шунтування вени Temporary vein bypass

Накладання затискачів на кінці вени Applying clamps on the end of the vein

Накладання затискача на дистальний відрізок вени Applying the clamp on the distal segment of the vein

34 / 200
Дівчинка 4-х років доставлена в лікарню зі скаргами на 3-кратне блювання за добу. Об’єктивно: запах ацетону з роту, млявість, блідість. Дані скарги виникли вперше після зловживання жирною, пряною їжею. В аналізі сечі ацетон +++. Що рекомендовано дитині по виписці з відділення в даному випадку? A 4-year-old girl was taken to the hospital with complaints of vomiting 3 times a day. Objectively: the smell of acetone from the mouth, lethargy, paleness. These complaints appeared for the first time after abusing fatty, spicy food. Acetone +++ in the urine analysis. What is recommended for the child upon discharge from the department in this case?

Лікування в спеціалізованому відділенні Treatment in a specialized department

Диспансерний нагляд протягом 5-ти років Dispensary supervision for 5 years

Протирецидивне лікування 1 раз на рік Anti-relapse treatment once a year

Тільки дієтотерапія протягом тижня Only diet therapy during the week

Дієтотерапія, диспансерний нагляд Dietotherapy, dispensary supervision

35 / 200
Хвора на дифузний токсичний зоб 30-ти років, не лікувалася. Після надмірного навантаження під час занять спортом виникли збудження, серцебиття, пітливість, блювання. Ps-140/хв., миготлива аритмія. АТ- 100/40 мм рт.ст. Тони серця звучні. Який найбільш імовірний діагноз? A 30-year-old patient with diffuse toxic goiter, was not treated. Excitement, palpitations, sweating, vomiting occurred after excessive exertion during sports. Ps-140/ min., atrial fibrillation. Blood pressure - 100/40 mm Hg. Heart sounds are sonorous. What is the most likely diagnosis?

Гіпоталамічний криз Hypothalamic crisis

Тиреотоксична криза Thyrotoxic crisis

Гостра серцево-судинна недостатність Acute cardiovascular failure

Гостра судинна недостатність Acute vascular insufficiency

Метаболічна кардіоміопатія Metabolic cardiomyopathy

36 / 200
Вагітна при терміні гестації 36 тижнів звернулася до лікаря жіночої консультації зі скаргами на нудоту, блювання, болі в епігастральній ділянці. При лабораторному обстеженні: гіпопротеїнемія, гіпербілірубінемія, тромбоцитопенія, підвищення трансаміназ. Яка основна тактика лікаря? A pregnant woman with a gestation period of 36 weeks turned to the doctor of the women's consultation with complaints of nausea, vomiting, pain in the epigastric area. During the laboratory examination: hypoproteinemia, hyperbilirubinemia, thrombocytopenia, increase in transaminases. What is the doctor's main tactic?

Консультація інфекціоніста Infectious disease consultation

Негайна госпіталізація у пологовий будинок Immediate hospitalization in the maternity hospital

Консультація терапевта Therapist consultation

Консультація хірурга Surgeon consultation

Консультація гастроентеролога Gastroenterologist consultation

37 / 200
Першовагітна 25-ти років з терміном гестації 26 тижнів звернулася до лікаря жіночої консультації зі скаргами на погіршення загального стану, спрагу, слабкість, блювання, сонливість, болі по всьому животу впродовж останніх 8-ми діб. Хворіє на цукровий діабет. Запах ацетону з рота. Ps-100/хв., АТ- 100/60 мм рт.ст. Рівень цукру в крові - 19,4 ммоль/л. Набряки ніжних кінцівок. Який найбільш імовірний діагноз? A 25-year-old first-time pregnant woman with a gestation period of 26 weeks turned to the doctor of the women's consultation with complaints of worsening general condition, thirst, weakness, vomiting, drowsiness, pains all over abdomen during the last 8 days. Suffers from diabetes. Acetone smell from the mouth. Ps-100/min., BP- 100/60 mm Hg. Blood sugar level - 19.4 mmol/l. Edema of tender limbs . What is the most likely diagnosis?

Кетонемічна кома Ketonemic coma

Гіпоглікемічна кома Hypoglycemic coma

Гіперлактацидемічна кома Hyperlactacidemic coma

Гіперосмолярна кома Hyperosmolar coma

Прееклампсія, тяжка форма Preeclampsia, severe form

38 / 200
Черговий лікар міської лікарні був викликаний у палату до пацієнта, що раптово втратив свідомість. При огляді в першу чергу необхідно: The doctor on duty of the city hospital was called to the ward to a patient who suddenly lost consciousness. During the examination, first of all, it is necessary:

Оглянути зіниці пацієнта Examine the patient's pupils

Пальпувати пульс на сонній артерії, візуально оцінити дихання, провести аускультацію серцевої діяльності Palpate the pulse on the carotid artery, visually assess breathing, conduct auscultation of cardiac activity

Визначити рівень глюкози в крові Determine blood glucose level

Реєстрація ЕКГ Registration of ECG

Вимірити АТ Measure blood pressure

39 / 200
Дитина 8-ми років доставлена в лікарню в непритомному стані. Протягом 5-ти днів хворіє на кишкову інфекцію з підвищенням температури, діареєю, блюванням. Різка дегідратація. Часте поверхневе дихання. Виражена неврологічна симптоматика: ністагм, менінгеальні знаки, гіпертонус м’язів, судоми. Запаху ацетону у повітрі немає. Глюкоза крові - 42 ммоль/л, кетонемія в нормі, кетонурія відсутня, осмолярність крові - 385 мосм/л. Який найбільш імовірний діагноз? An 8-year-old child was brought to the hospital in an unconscious state. For 5 days he has been suffering from an intestinal infection with fever, diarrhea, vomiting. Severe dehydration. Frequent shallow breathing. Pronounced neurological symptoms: nystagmus, meningeal signs, muscle hypertonus, convulsions. There is no smell of acetone in the air. Blood glucose is 42 mmol/l, ketonemia is normal, there is no ketonuria, blood osmolarity is 385 mosm/l. What is the most probable diagnosis?

Гіперосмолярна кома Hyperosmolar coma

Кетоацидотична кома Ketoacidotic coma

Менінгоенцефальний синдром Meningoencephalic syndrome

Церебральна кома Cerebral coma

Лактатацидемічна кома Lactatacidemic coma

40 / 200
Хворий 65-ти років скаржиться на зміну кольору сечі. Сеча червона, з домішками кров’яних згустків, що нагадують 'черв’яків' болю немає, непокоїть слабкість. У сечі: гематурія. У крові: Hb- 84 г/л, ШОЕ- 31 мм/год. Який найбільш імовірний діагноз? A 65-year-old patient complains of a change in the color of urine. The urine is red, with admixtures of blood clots, reminiscent of 'worms'; there is no pain, weakness worries . In the urine: hematuria. In the blood: Hb - 84 g/l, ESR - 31 mm/h. What is the most likely diagnosis?

Пухлина нирки Kidney tumor

Гострий цистит Acute cystitis

Гострий пієлонефрит Acute pyelonephritis

Сечокам’яна хвороба Urolithiasis

Гострий гломерулонефрит Acute glomerulonephritis

41 / 200
На прийомі в жіночій консультації вагітна 37 тижнів поскаржилася на різкий головний біль, погіршення зору, біль у ділянці сонячного сплетіння. Об’єктивно: АТ- 170/110 мм рт.ст., набряки нижніх кінцівок. Яку першу невідкладну допомогу слід надати у жіночій консультації? At the appointment at the women's consultation, a 37-week pregnant woman complained of a sharp headache, visual impairment, pain in the solar plexus area. Objectively: BP- 170/110 mm mercury, swelling of the lower extremities. What first aid should be given in a woman's consultation?

Введення лазіксу 40 мг внутрішньовенно Lasix 40 mg IV

Введення кокарбоксилази 150 мг внутрішньовенно Introduction of cocarboxylase 150 mg intravenously

Введення магнію сульфіту 25% -20,0, сибазону 0,02 внутрішньовенно, госпiталiзацiя до акушерського стаціонару Introduction of magnesium sulfite 25% -20.0, sibazone 0.02 intravenously, hospitalization in an obstetric hospital

Введення аскорбінової кислоти 5% -5 мл внутрішньовенно Introduction of ascorbic acid 5% -5 ml intravenously

Введення анальгіну 50% - 2,0 Introduction of analgin 50% - 2.0

42 / 200
Хлопчик 12-ти років хворіє на атопічну бронхіальну астму з важким перебігом. Під час останнього нападу 4 інгаляції сальбутамолу ефекту не дали. Наросли задишка, тахікардія, неспокій. Шкіра бліда з ціанотичним відтінком. У легенях різко ослаблене дихання, хрипи не вислуховуються. Який з перелічених заходів є першочерговим? A 12-year-old boy suffers from atopic bronchial asthma with a severe course. During the last attack, 4 inhalations of salbutamol had no effect. Shortness of breath, tachycardia, restlessness developed. Skin she is pale with a cyanotic shade. Breathing is sharply weakened in the lungs, wheezing is not heard. Which of the listed measures is the first priority?

Внутрішньовенне введення еуфіліну Intravenous Euphilin

Інгаляція зволоженого кисню Inhalation of humidified oxygen

Внутрішньовенне введення кларитроміцину Intravenous administration of clarithromycin

Внутрішньовенне введення амброксолу Intravenous administration of ambroxol

Внутрішньовенне введення преднізолону Intravenous administration of prednisone

43 / 200
Хворий скаржиться на головний біль, біль у ділянці серця, нудоту, сухість у роті, безсоння. Об’єктивно: обличчя гіперемоване, тремор пальців рук і язика, хода некоординована, виражена пітливість, настрій різко знижений. Зловживає спиртними напоями, останню добу алкоголь не вживав. Визначте психопатологічний синдром: The patient complains of a headache, heart pain, nausea, dry mouth, insomnia. Objectively: hyperemic face, tremors of fingers and tongue, gait uncoordinated, pronounced sweating, mood sharply lowered. Abuses alcoholic beverages, has not consumed alcohol in the last day. Define psychopathological syndrome:

Астенічний Asthenic

Деліріозний Delirious

Абстинентний Abstinent

Депресивний Depressed

Аментивний Amentary

44 / 200
У дівчини 19-ти років після зґвалтування виникли постійна плаксивість, тривога, настрій став пригніченим, знизилася працездатність, рухи загальмовані, з’явилися думки про недоцільність життя. Визначте психопатологічний стан: After the rape, a 19-year-old girl developed constant tearfulness, anxiety, her mood became depressed, her work capacity decreased, her movements were inhibited, and thoughts about the futility of life appeared. Define psychopathological condition:

Тривожна депресія Anxious Depression

Судинна депресія Vascular depression

Інволюційна депресія Involutional depression

Реактивна депресія Reactive Depression

Ендогенна депресія Endogenous depression

45 / 200
У жінки 32-х років після внутрішньовенної ін’єкції антибіотиків пеніцілінового ряду з приводу загострення хронічного сальпінгоофоріту раптово з’явилась задишка, біль в ділянці серця, слабкість, свербіж шкіри обличчя та верхніх кінцівок, висипка. Об’єктивно: свідомість запаморочена, АТ- 80/50 мм рт.ст., пульс ниткоподібний, серцева аритмія. Попередній діагноз - анафілактичний шок. Які перші дії? A 32-year-old woman suddenly developed shortness of breath, heart pain, weakness, itching after an intravenous injection of penicillin antibiotics due to exacerbation of chronic salpingo-oophoritis of the skin of the face and upper limbs, a rash. Objectively: consciousness is dizzy, blood pressure - 80/50 mm Hg, the pulse is filamentous, cardiac arrhythmia. The preliminary diagnosis is anaphylactic shock. What are the first actions?

Ввести антигістамінні препарати Enter antihistamines

Провести інтубацію трахеї Perform tracheal intubation

Накласти джгут проксімальніше місця ін’єкції Apply a tourniquet proximal to the injection site

Ввести розчин адреналіну Enter adrenaline solution

Розпочати інфузійну терапію Start infusion therapy

46 / 200
Хвора 46-ти років скаржиться на появу протягом двох тижнів випорожнень кров’ю більш за 12 разів на добу, болю в животі, суглобах; болючі висипки у порожнині рота та нижніх кінцівках, набряк колінних та гомілковостопних суглобів, слабкість, підвищення температури тіла понад 39oC, втрату ваги тіла. Який метод обстеження є найбільш інформативним? A 46-year-old patient complains of the appearance of bloody stools more than 12 times a day, pain in the abdomen, joints, painful rashes in the oral cavity for two weeks and lower limbs, swelling of knee and ankle joints, weakness, increase in body temperature above 39oC, loss of body weight. Which examination method is the most informative?

Рентгенографія суглобів X-ray of joints

Загальний аналіз крові General blood test

Біопсія кишечнику та шкіри Intestinal and skin biopsy

Рентгенологічне дослідження кишечнику X-ray examination of intestines

Ендоскопічне дослідження Endoscopy examination

47 / 200
Під час діагностичної ендоскопії у хворого 45-ти років виникла асистолія, яка діагностована електрокардіографічно. З яких заходів слід негайно розпочати лікування? During a diagnostic endoscopy, a 45-year-old patient developed asystole, which was diagnosed electrocardiographically. What measures should be taken to treat it immediately?

Електрокардіостимуляція Electronic cardiostimulation

Інфузія гідрокарбонату натрію Sodium bicarbonate infusion

Масаж серця, ШВЛ, адреналін Heart massage, ventilator, adrenaline

Дефібриляція Defibrillation

Введення атропіну Atropine administration

48 / 200
У дитини 10-ти років зупинена артеріальна кровотеча (плечова артерія) через 10 хвилин після травми. При надходженні АТ- 90/50 мм рт.ст., еритроцити - 2,5•1012/л, Hb- 60 г/л. Раніше при переливанні цільної крові була анафілактоїдна реакція. Який препарат крові найбільш безпечний для корекції крововтрати? A 10-year-old child has stopped arterial bleeding (brachial artery) 10 minutes after the injury. On admission, blood pressure is 90/50 mm Hg, erythrocytes - 2.5•1012/l, Hb- 60 g/l. Previously, there was an anaphylactoid reaction during transfusion of whole blood. Which blood preparation is the safest for blood loss correction?

Еритроцитарна маса Erythrocyte mass

Свiжозаморожена плазма Fresh-frozen plasma

Відмиті еритроцити Washed erythrocytes

Лейкоконцентрат Leukoconcentrate

Цільна кров Whole blood

49 / 200
Дитину 11-ти місяців на 3-й день хвороби госпіталізовано до інфекційного стаціонару з підвищенням температури тіла до 38oC, багаторазовим блюванням і частими водянистими випорожненнями. Маса тіла знижена на 6%. Який метод регідратації треба призначити? On the 3rd day of illness, an 11-month-old child was hospitalized in an infectious disease hospital with an increase in body temperature to 38oC, repeated vomiting and frequent watery stools. Body weight decreased by 6%. What method of rehydration should be prescribed?

Введення рідини ендогастрально Endogastric fluid administration

Введення рідини підшкірно крапельно Subcutaneous drip

Оральна регідратація Oral rehydration

Внутрішньовенний крапельний Intravenous drip

Внутрішньовенний струминний Intravenous jet

50 / 200
Хворий 17-ти років 3 дні хворіє на ГРЗ. Після різкого вставання з ліжка відчув загальну слабкість, шум у вухах, потемніння в очах. Шкіра холодна, бліда, з мармуровим відтінком. АТ- 90/60 мм рт.ст., ЧСС- 56/хв. Який найбільш імовірний діагноз? A 17-year-old patient has been suffering from acute respiratory syndrome for 3 days. After suddenly getting out of bed, he felt general weakness, tinnitus, darkening of the eyes. The skin is cold, pale, with a marble shade. Blood pressure - 90/60 mm Hg, heart rate - 56/min. What is the most likely diagnosis?

Інфекційно-токсичний шок Infectious-toxic shock

Ортостатичний колапс Orthostatic collapse

Енцефаліт Encephalitis

Непритомність Fainting

Синдром Морганьї-Адамса-Стокса Morganhi-Adams-Stokes syndrome

51 / 200
Хворий 66-ти років скаржиться на серцебиття, біль у серці, слабкість. Симптоми з’явилися раптово, після фізичного навантаження. На ЕКГ: комплекси QRS поширені до 0,12 секунд, зубець P перед шлуночковим комплексом відсутній, ЧСС- 200/хв. Який препарат є препаратом вибору в цьому випадку? A 66-year-old patient complains of palpitations, heart pain, weakness. The symptoms appeared suddenly, after physical exertion. On the ECG: QRS complexes spread to 0 ,12 seconds, the P wave before the ventricular complex is absent, the heart rate is 200/min. What drug is the drug of choice in this case?

Обзидан Obzydan

Хінідин Quinidine

Лідокаїн Lidocaine

Дигоксин Digoxin

Верапаміл Verapamil

52 / 200
Після введення ампіциліну у хворої на сальпінгіт, в області рук з’явились еритематознобульозні висипки, шкіра відторгається значними шарами по типу рукавичок. Який найбільш імовірний діагноз? After administration of ampicillin in a patient with salpingitis, erythematous bullous rashes appeared in the hands, the skin is rejected in significant layers like gloves. What is the most likely diagnosis?

Синдром Лайела Lyell Syndrome

Генералізований кандидоз Generalized candidiasis

Епідермофітія Epidermophyta

Алергічний дерматит Allergic dermatitis

Екзема Eczema

53 / 200
Хворий 45-ти років поступив у відділення зі скаргами на пекучий біль, сильне розпирання тканин пальців обох стоп. Об’єктивно: пухирі, наповнені геморагічною рідиною; після зняття пухирів видно некротизовану шкіру, котра нечутлива і неболюча. Який найбільш імовірний діагноз? A 45-year-old patient was admitted to the department with complaints of burning pain, severe swelling of the tissues of the fingers of both feet. Objectively: blisters filled with hemorrhagic fluid; after removal blisters show necrotic skin, which is insensitive and painless. What is the most likely diagnosis?

Відмороження ІІ ступеня Defrosting of the 2nd degree

Відмороження ІІІ ступеня III degree frostbite

Термічний опік ІІІ А ступеня Thermal burn III A degree

Відмороження І ступеня Defrosting of the 1st degree

Термічний опік ІІ ступеня II degree thermal burn

54 / 200
У 6-місячної дитини несподівано з’явились занепокоєність, блювання, відходження з каловими масами крові у вигляді 'малинового желе'. При обстеженні живота виявлені напруження черевної стінки, праворуч від пупка -ковбасоподібне утворення. Який діагноз найбільш імовірний? A 6-month-old child suddenly developed anxiety, vomiting, and passing fecal masses of blood in the form of 'raspberry jelly'. Abdominal examination revealed tension in the abdominal wall, to the right of the navel - a sausage-like formation. What is the most likely diagnosis?

Кишкова інвагінація Intestinal intussusception

Геморагічний васкуліт Hemorrhagic vasculitis

Дизентерія Dysentery

Ентероколіт Enterocolitis

Поліп кишечника Intestinal polyp

55 / 200
Дитина 2 роки, хворіє протягом 3-х днів. Захворювання розпочалося з підвищення температури тіла до 38,0oC, блюванням, через 4 години приєдналася діарея. Випорожнення рідкі водянисті 8 разів на добу, домішки слизу та крові відсутні. На час огляду дитина активна, шкіра чиста, тургор тканин незмінений, слизові вологі, п’є жадібно, останній час мочилася 1 годину тому. Тактика ведення хворого: The child is 2 years old, has been sick for 3 days. The disease began with an increase in body temperature to 38.0oC, vomiting, after 4 hours diarrhea joined. The stools are liquid and watery 8 times a day, there are no impurities of mucus and blood. At the time of the examination, the child is active, the skin is clean, the turgor of the tissues is unchanged, the mucous membranes are moist, he drinks greedily, the last time he urinated was 1 hour ago. Patient management tactics:

Планова госпіталізація через 1 тиждень Planned hospitalization in 1 week

Проведення оральної регідратації в домашніх умовах Performing oral rehydration at home

Проведення парентеральної регідратації в домашніх умовах Performing parenteral rehydration at home

Негайна госпіталізація Immediate hospitalization

- -

56 / 200
Чоловіка покусав сусідський собака за стегна та передпліччя біля 20 хвилин тому назад. Які засоби повинен застосувати сімейний лікар на догоспітальному етапі? A man was bitten by a neighbor's dog on his thighs and forearms about 20 minutes ago. What measures should the family doctor use at the pre-hospital stage?

Промити рану розчином господарського мила Wash the wound with a solution of household soap

Промити рану розчином йоду Wash the wound with iodine solution

Ввести протиправцеву сироватку Enter tetanus serum

Промити рану антибіотиками Wash the wound with antibiotics

Накласти герметичну пов’язку Apply an airtight bandage

57 / 200
У хворого із вкрай важким ступенем кістковомозкової форми гострої променевої хвороби у початковому періоді з’явилися: лихоманка, сильний головний біль, адинамія, біль у м’язах і суглобах. Який синдром гострої променевої хвороби у хворого? A patient with an extremely severe bone marrow form of acute radiation sickness in the initial period developed: fever, severe headache, adynamia, pain in muscles and joints What is the patient's syndrome of acute radiation sickness?

Синдром ураження нервової системи Syndrome of damage to the nervous system

Гастроінтестинальний синдром Gastrointestinal syndrome

Геморагічний синдром Hemorrhagic syndrome

Синдром загальної токсемії Syndrome of general toxemia

Синдром інфекційних ускладнень Syndrome of infectious complications

58 / 200
Повторновагітна 28-ми років з приводу теперішньої вагітності у жіночій консультації не спостерігалася. З анамнезу: під час попередньої вагітності виявлений гестаційний цукровий діабет. Скарги на шкірний свербіж, пітливість, загальну слабкість. З рота відзначається запах ацетону. Тиждень тому при проведенні ультразвукового обстеження діагностована вагітність 10 тижнів. Яка тактика лікаря? A 28-year-old re-pregnant woman was not observed in the women's consultation regarding the current pregnancy. From the anamnesis: gestational diabetes was detected during the previous pregnancy. Complaints of skin itching, sweating , general weakness. The smell of acetone is noted from the mouth. A week ago, during an ultrasound examination, a 10-week pregnancy was diagnosed. What are the doctor's tactics?

Направити до гінекологічного відділення Send to gynecological department

Рекомендувати вагітній стати на облік у жіночу консультацію Recommend a pregnant woman to register at a women's consultation

Амбулаторне обстеження Outpatient examination

Госпіталізація в ендокринологічне відділення в ургентному порядку Hospitalization in the endocrinology department in an urgent order

Планова госпіталізація в ендокринологічне відділення Planned hospitalization in the endocrinology department

59 / 200
До приймального відділення доставлено чоловіка 30-ти років після утоплення у прісній воді, яке відбулося близько 40 хвилин тому і супроводжувалося клінічною смертю та успішною реанімацією. Чоловік у свідомості, при аускультації у легенях вислуховуються хрипи над всіма легеневими полями. Основний напрямок подальшої терапії: A 30-year-old man was brought to the reception department after drowning in fresh water, which occurred about 40 minutes ago and was accompanied by clinical death and successful resuscitation. The man is conscious, upon auscultation in the lungs, rales are heard over all lung fields. The main direction of further therapy:

Відновлення ОЦК Recovery of BCC

Киснетерапія Oxygen therapy

Дегідратація Dehydration

Введення глюкокортикоїдів Introduction of glucocorticoids

Введення інгібіторів протеолізу Introduction of proteolysis inhibitors

60 / 200
У роділлі 35-ти років відбулися треті термінові нормальні пологи. Загальна крововтрата склала 400 мл, через 1 годину з піхви з’явилося 100 мл кров’янистих виділень із згустками. Які діагностичні заходи необхідно провести для уточнення діагнозу? A 35-year-old woman gave birth to a third urgent normal delivery. The total blood loss was 400 ml, after 1 hour 100 ml of bloody discharge appeared from the vagina clots. What diagnostic measures should be carried out to clarify the diagnosis?

Кюретаж порожнини матки Curettage of the uterine cavity

Загальний аналіз крові General blood test

Визначення висоти стояння дна матки Determining the standing height of the uterine fundus

Зондування порожнини матки Probing of the uterine cavity

Огляд шийки матки в дзеркалах Examination of the cervix in mirrors

61 / 200
У хворої 36-ти років відбулися патологічні пологи на фоні прееклампсії. В анамнезі вірусний гепатит A 36-year-old patient had a pathological delivery against the background of preeclampsia. She has a history of viral hepatitis

Гепатит в анамнезі History of hepatitis

Гострий внутрішньосудинний гемоліз 6- Гіпертензія Acute intravascular hemolysis 6- Hypertension

Післягеморагічна анемія Posthemorrhagic anemia

Після гіпотонії матки, кровотечі та трансфузії еритроцитарної маси іншої групи з’явилися наступні ознаки: іктеричність шкіри та склер, гематурія, геморагії, лихоманка, болі в попереку. Назвіть головну причину цих симптомів: After hypotonia of the uterus, bleeding and transfusion of erythrocyte mass of another group, the following symptoms appeared: icterus of the skin and sclera, hematuria, hemorrhages, fever, back pain. Name the main cause of these symptoms:

Гострий пієлонефрит Acute pyelonephritis

62 / 200
У 14-річного хлопця з верифікованою виразковою хворобою відмічається посилення больового синдрому. Біль постійний, не зменшується після прийому їжі, відмічається стул чорного кольору. При обстеженні живіт різко болючий при пальпації з іррадіацією болю в ліве підребер’я, відмічається напруження м’язів живота, в пілородуоденальній зоні пальпується щільне утворення. Яка найбільш обґрунтована тактика ведення хворого? A 14-year-old boy with a verified peptic ulcer has an increased pain syndrome. The pain is constant, does not decrease after eating, black stool is noted. Abdominal examination is very painful upon palpation with pain radiating to the left hypochondrium, tension of the abdominal muscles is noted, a dense mass is palpated in the pyloroduodenal zone. What is the most justified tactic for managing the patient?

Симптоматичне лікування вдома Symptomatic treatment at home

Обстеження та спостереження в умовах поліклініки Examination and observation in polyclinic conditions

Планова госпіталізація в хірургічне відділення Planned hospitalization in the surgical department

Госпіталізація в гастроентерологічне відділення Hospitalization in the gastroenterology department

Ургентна госпіталізація хворого в хірургічне відділення Urgent hospitalization of the patient in the surgical department

63 / 200
Хворий 30-ти років скаржиться на пронос з домішками слизу і крові, переймоподібний біль у животі. Хворіє 4 роки, впродовж яких схуд на 10 кг. Об’єктивно: Ps- 100/хв., АТ100/60 мм рт.ст., температура - 38оС. Живіт м’який, болючий у лівій здухвинній ділянці. У крові: ер.- 3,1•1012/л, НЬ- 85 г/л, лейк.- 11,2•109/л, ШОЕ- 32 мм/год. Реакція Грегерсена позитивна. Іригоскопія - товста кишка звужена, зернистість слизової оболонки, гаустри відсутні, контури нечіткі. Який найбільш імовірний діагноз? A 30-year-old patient complains of diarrhea with impurities of mucus and blood, spasm-like abdominal pain. He has been ill for 4 years, during which he lost 10 kg. Objectively : Ps- 100/min., BP 100/60 mm Hg, temperature - 38°C. The abdomen is soft, painful in the left iliac region. In the blood: er.- 3.1•1012/l, НБ- 85 g /l, leuk.- 11.2•109/l, ESR- 32 mm/h. Gregersen's reaction is positive. Irigoscopy - the colon is narrowed, the mucous membrane is granular, haustra are absent, the contours are unclear. What is the most likely diagnosis?

Хвороба Крона Crohn's disease

Синдром подразненої товстої кишки Irritable colon syndrome

Неспецифічний виразковий коліт Nonspecific ulcerative colitis

Хронічний коліт Chronic Colitis

Туберкульоз кишечнику Intestinal tuberculosis

64 / 200
Чоловік 45-ти років на міському пляжі у спекотну погоду раптово втратив свідомість. Об’єктивно: зіниці звужені, реакцій на легкі подразники немає, шкіра бліда, температура тіла - 39оС, частота дихань - 28/хв., АТ-100/60 мм рт.ст., пульс ниткоподібний, ритмічний, 100 /хв. Спостерігаються періодичні судоми кінцівок. Яка причина вказаного стану? A 45-year-old man suddenly lost consciousness on a city beach in hot weather. Objectively: the pupils are narrowed, there are no reactions to light stimuli, the skin is pale, the body temperature - 39oC, respiratory rate - 28/min., BP-100/60 mm Hg, pulse thread-like, rhythmic, 100/min. Periodic convulsions of the limbs are observed. What is the cause of the specified condition?

Гостре порушення мозкового кровообігу Acute cerebrovascular accident

Анафілактичний шок Anaphylactic shock

Сонячний удар Sunstroke

Гострий інфаркт міокарду Acute myocardial infarction

Теплова непритомність Heat syncope

65 / 200
У чоловіка 30-ти років в ході огляду в поліклініці терапевтом виявлено ножове поранення правої половини живота. Живіт м’який, біль відсутній. В крові: НЬ-140 г/л, лейк.6,8•109/л. Яку тактику слід обрати лікарю? A 30-year-old man was examined in a polyclinic by a therapist who discovered a stab wound to the right half of the abdomen. The abdomen is soft, there is no pain. In the blood: НБ-140 g/l, leuk.6.8•109/l. What tactics should the doctor choose?

Направити постраждалого до хірургічного стаціонару Send the victim to a surgical hospital

Направити на планове лікування до хірургічного відділення Send for planned treatment to the surgical department

Призначити рентгенографію органів черевної порожнини Prescribe X-ray of abdominal organs

Призначити УЗД органів черевної порожнини Prescribe abdominal ultrasound

Амбулаторне лікування Outpatient treatment

66 / 200
Хворий 38-ми років, що страждає на нирковий туберкульоз, в непритомному стані привезений родичами до районної лікарні. Об’єктивно: кома 1. Гіпотрофія 2 ст. Шкіра та слизові оболонки коричневого кольору з бронзовим полиском. На шкірі спостерігаються окремі осередки вітиліго. АТ-65/0 мм рт.ст., ЧСС- 44/хв. Діагностована гостра надниркова недостатність. Який препарат треба ввести в першу чергу? A 38-year-old patient suffering from renal tuberculosis was brought by relatives to the district hospital in an unconscious state. Objectively: coma 1. Hypotrophy 2nd stage. Skin and brown mucous membranes with a bronze sheen. Individual foci of vitiligo are observed on the skin. BP-65/0 mm Hg, heart rate- 44/min. Acute adrenal insufficiency is diagnosed. What drug should be administered first?

Новодрин Novodrin

Реополіглюкін Rheopoliglyukin

Адреналін Adrenaline

Дексаметазон Dexamethasone

Мезатон Mesaton

67 / 200
У вагітної жінки у терміні 36 тижнів раптово з’явився біль в епігастральній ділянці, блювання кавовою гущею. Після обстеження встановлений діагноз: вагітність 1, 36 тижнів, прееклампсія важкого ступеня, HELLP-синдром. Тактика ведення вагітної: At 36 weeks, a pregnant woman suddenly developed pain in the epigastric area, vomiting coffee grounds. After the examination, the diagnosis was established: pregnancy 1, 36 weeks, severe preeclampsia degree, HELLP-syndrome. Tactics of managing a pregnant woman:

Гемостатична терапія Hemostatic therapy

Термінове оперативне розродження Urgent operative delivery

Стимуляція пологової діяльності Stimulation of labor activity

Дезінтоксикаційна терапія Detoxification therapy

Інтенсивна терапія прееклампсії Intensive therapy of preeclampsia

68 / 200
Жінці 28-ми років виконано кесарів розтин за показаннями слабкості пологової діяльності. Під час операції крововтрата склала 1200 мл та була негайно відновлена переливанням еритроцитарної маси у кількості 350 мл, плазми - 800 мл, кристалоїдів -1500 мл. Після операції виведена сеча у кількості 150 мл бурого кольору. При лабораторному дослідженні у сечі кров’яний детрит. Яке виникло ускладнення? A 28-year-old woman underwent a caesarean section on the grounds of weakness in labor. During the operation, blood loss amounted to 1200 ml and was immediately restored by transfusion of erythrocyte mass in the amount of 350 ml, plasma - 800 ml, crystalloids - 1500 ml. After the operation, urine in the amount of 150 ml was brown. During the laboratory examination, there was blood in the urine. What complication occurred?

Ушкодження сечоводу Damage to the ureter

Переливання несумісної крові Transfusion of incompatible blood

Ушкодження сечового міхура Bladder damage

Ускладнень немає No complications

Характеризує початок розвитку ДВЗ-синдрому Characterizes the beginning of the development of DVZ-syndrome

69 / 200
Хлопчик 8-ми років впав з велосипеда, відчув різкий біль у шийному відділі. Об’єктивно: голова повернута вправо, спроби повороту голови вліво болісні. Для транспортної імобілізації необхідно: An 8-year-old boy fell off a bicycle, felt a sharp pain in the neck. Objectively: the head is turned to the right, attempts to turn the head to the left are painful. For transport immobilization necessary:

Виведення голови в середнє положення Bringing the head to the middle position

Парентеральне введення міорелаксантів Parenteral administration of muscle relaxants

Фіксація голови у цьому ж положенні Fixing the head in the same position

Гіперкорекція (нахил голови у протилежний бік) Hypercorrection (head tilt to the opposite side)

Необхідності у фіксації немає There is no need for fixation

70 / 200
Селективні а-адреноміметики недоцільно застосовувати для поновлення серцевої діяльності внаслідок: Selective α-adrenomimetics should not be used to restore cardiac activity due to:

Вираженого аритмогенного ефекту Pronounced arrhythmogenic effect

Відсутності позитивного ефекту на міокард Absence of a positive effect on the myocardium

Здатності підвищувати систолічний артеріальний тиск Ability to increase systolic blood pressure

Нездатності підвищувати діастолічний артеріальний тиск Inability to increase diastolic blood pressure

Значного підвищення потреби міокарда у кисні Significantly increased myocardial oxygen demand

71 / 200
У хворого після важкої скелетної травми на другу добу раптово виникло зниження артеріального тиску, порушення свідомості та самостійного дихання, дрібноточкові крововиливи на шкірі. Який найбільш імовірний діагноз? On the second day after a severe skeletal injury, the patient suddenly experienced a decrease in blood pressure, impaired consciousness and spontaneous breathing, and small point hemorrhages on the skin. What is the most likely diagnosis?

Септичний шок Septic shock

Синдром тривалого стиснення Prolonged compression syndrome

Жирова емболія Fat embolism

Анафілактичний шок Anaphylactic shock

Травматичний шок Traumatic shock

72 / 200
В аеропорт прибув літак із ендемічної по холері країни. Лікар санітарно-карантинного пункту аеропорту при огляді літака виявив хворого громадянина України з діареєю. Пізніше був підтверджений діагноз холера. Особи, які знаходились в контакті з хворим на холеру підлягають: A plane from a cholera-endemic country arrived at the airport. The doctor of the sanitary quarantine station of the airport, during the inspection of the plane, found a sick citizen of Ukraine with diarrhea. Later, the diagnosis of cholera was confirmed. Persons , who were in contact with a cholera patient are subject to:

Розміщенню в ізоляторі з обов’язковим проведенням превентивного лікування Placing in an isolation ward with mandatory preventive treatment

Медичному нагляду протягом 5-ти діб Medical supervision within 5 days

Госпіталізації у провізорний стаціонар Hospitalizations in a provisional hospital

Госпіталізації в інфекційний стаціонар Hospitalizations in an infectious hospital

Ізоляції вдома Isolations at home

73 / 200
Для профілактики гнійно-септичних ускладнень при постановці катетера у центральні та периферійні вени потрібно: For the prevention of purulent-septic complications when placing a catheter in the central and peripheral veins, you need:

Введення тромболітичних препаратів Introduction of thrombolytic drugs

Щоденний догляд та асептичні перев’язки Daily care and aseptic dressings

Зрошення розчином антибіотиків місця знаходження катетера Irrigation of the catheter site with antibiotic solution

Не застосовувати білкових препаратів Do not use protein preparations

Застосовувати при інфузії автоматичні дозатори Use automatic dispensers during infusion

74 / 200
Хлопчик 2-х років, що страждає на гемофілію А, доставлений в клініку із скаргами на різкий біль в животі, наростаючу блідість. Об’єктивно: стан дуже важкий, блідий, збудливість змінюється адинамією, виражена задишка: ЧД- 80/хв., ЧСС- 170/хв., АТ- 70/25 мм рт.ст. Живіт напружений, гази не виходять, сечовипускань не було. В крові: Hb- 55 г/л, лейкоцити - 5,2*109/л, ШОЕ-10 мм/год, центральний венозний тиск - 0 мм рт.ст., систолічний індекс - 1,5 л/м2 за хвилину. Лікар встановив наявність геморагічного шоку. Призначте невідкладну терапію: A 2-year-old boy suffering from hemophilia A was brought to the clinic with complaints of sharp abdominal pain, increasing pallor. Objectively: the condition is very serious , pale, excitability is replaced by adynamia, pronounced shortness of breath: BH - 80/min., HR - 170/min., BP - 70/25 mm Hg. Abdomen is tense, gases do not come out, there was no urination. In the blood: Hb- 55 g/l, leukocytes - 5.2*109/l, ESR - 10 mm/h, central venous pressure - 0 mm Hg, systolic index - 1.5 l/m2 per minute. The doctor established the presence of hemorrhagic shock . Prescribe emergency therapy:

Трансфузія еритроцитарної маси Transfusion of erythrocyte mass

Введення кріопреципітату Introduction of cryoprecipitate

Гемотрансфузія Hemotransfusion

Інфузійна терапія для підвищення ОЦК Infusion therapy to increase BCC

Трансфузія сухої плазми Transfusion of dry plasma

75 / 200
Потерпілого 33-х років через 3 години після землетрусу доставлено до клініки. Об’єктивно: відірвана верхня права кінцівка, кукса під джгутом. АТ-60/30 мм рт.ст., ЧСС140 /хв. Який ступінь шоку? A 33-year-old victim was brought to the clinic 3 hours after the earthquake. Objectively: the upper right limb was torn off, the stump was under a harness. AT-60/30 mm Hg, heart rate 140/min. What is the degree of shock?

IV IV

І And

ІІІ ІІІ

V V

ІІ II

76 / 200
Вагітна 40-ка років страждає на персистуючу бронхіальну астму, легкого перебігу. Хворіє на бронхіальну астму з 7-ми років. Вагітність VI, передують 4 пологи, госпіталізована до палати патології вагітних. Виник важкий напад бронхіальної астми (тахіпное більше 30 /хв., ЧСС більше 120/хв., різниця систолічного АТ на вдосі і видосі більше 18 мм рт.ст. ). Інгаляції сальбутамолу неефективні протягом 6-ти годин. Яка тактика лікаря? A 40-year-old pregnant woman suffers from persistent bronchial asthma, of a mild course. She has suffered from bronchial asthma since the age of 7. Pregnancy VI, preceded by 4 deliveries, hospitalized until wards of the pathology of pregnant women. A severe attack of bronchial asthma occurred (tachypnea more than 30/min., heart rate more than 120/min., difference in systolic blood pressure between inspiration and expiration more than 18 mm Hg). Salbutamol inhalations are ineffective for 6 hours. What doctor's tactics?

Внутрішньовенно ввести преднізолон 60 мг - 90 мг, повторюючи введення по 30 мг кожні 3-4 години до поліпшення стану Intravenously introduce prednisone 60 mg - 90 mg, repeating the introduction of 30 mg every 3-4 hours until the condition improves

Внутрішньовенно галоперидол, седативні препарати Intravenous haloperidol, sedatives

Внутрішньовенно ввести платифілін, повторюючи кожні 3-4 години до поліпшення стану Inject platyphylline intravenously, repeating every 3-4 hours until improvement

Внутрішньовенно крапельно антибіотики широкого спектру дії Intravenous drip of broad-spectrum antibiotics

Внутрішньовенно крапельно розчин анаприліну кожні 3-4 години до поліпшення стану Intravenous drip solution of anaprilin every 3-4 hours until the condition improves

77 / 200
Сімейного лікаря викликали до хлопчика 5-ти років, що хворіє на епілепсію. За 3 дні до того батьки самостійно відмінили протисудомні препарати. Об’єктивно у дитини клонікотонічні генералізовані судоми, свідомість відсутня. Шкіра бліда, волога. Дихання аритмічне, тахікардія. Зіниці звужені. Що в першу чергу повинен зробити лікар? The family doctor was called to see a 5-year-old boy suffering from epilepsy. 3 days before that, the parents independently discontinued the anticonvulsant drugs. Objectively, the child has generalized clonicotonic convulsions, unconsciousness. The skin is pale, moist. Breathing is arrhythmic, tachycardia. Pupils are narrowed. What should the doctor do first?

Призначити серцеві глікозиди Prescribe cardiac glycosides

Ввести дихальні аналептики Enter respiratory analeptics

Ввести протисудомні препарати парентерально Introduce anticonvulsant drugs parenterally

Викликати машину швидкої медичної допомоги Call an ambulance

Проводити серцево-легеневу реанімацію Perform cardiopulmonary resuscitation

78 / 200
У молодого чоловіка під час акту дефекації при натужуванні раптово виник гострий біль у лівій половині грудної клітки, задишка, запаморочення. Легеневий анамнез відсутній. Доставлений в стаціонар через 2 години. Блідий, адинамічний, задишка більше 35 за хвилину. Ps- 110/хв., АТ- 90/60 мм рт.ст. Лабораторні дані: гематокрит 26%. Дихання зліва не проводиться, перкуторно - вкорочення перкуторного звуку. Рентгенологічно: масивне затемнення з горизонтальним рівнем. При пункції отримано більше ніж 500 мл крові. Кров продовжує поступати. Який найбільш імовірний діагноз? A young man suddenly developed sharp pain in the left half of the chest, shortness of breath, and dizziness during the act of defecation when straining. There is no pulmonary history. He was taken to the hospital 2 hours later Pale, adynamic, shortness of breath more than 35 per minute. Ps- 110/min., BP- 90/60 mm Hg. Laboratory data: hematocrit 26%. Breathing is not performed on the left, percussion - shortening of the percussion sound. X-ray: massive darkening with a horizontal level. More than 500 ml of blood was obtained during the puncture. Blood continues to flow. What is the most likely diagnosis?

Пневмоторакс Pneumothorax

Гострий ексудативний плеврит Acute exudative pleurisy

Напад стенокардії Attack of angina pectoris

Спонтанний гемоторакс Spontaneous hemothorax

Гострий інфаркт міокарда Acute myocardial infarction

79 / 200
Пацієнт під час ковтання цукерки захрипів, схопився за шию, посинів і через 3 хвилини знепритомнів. Об’єктивно: свідомість відсутня, обличчя і шия ціанотичні, нерегулярні та непродуктивні рухи грудної клітки. На сонних артеріях - слабка пульсація. При пальцевій ревізії ротоглотки визначається округле стороннє тіло, змістити яке не вдається. Спроба змістити його при перекиданні пацієнта через коліно - невдала. Які подальші дії? While swallowing a candy, the patient snored, grabbed his neck, turned blue and fainted after 3 minutes. Objectively: consciousness is absent, the face and neck are cyanotic, irregular and unproductive movements of the chest. There is a weak pulsation on the carotid arteries. During a finger inspection of the oropharynx, a rounded foreign body is identified, which cannot be dislodged. An attempt to dislodge it while rolling the patient over the knee is unsuccessful. What are the further actions?

Пункція трахеї Trache puncture

Нижня трахеотомія Lower tracheotomy

Крікотиреотомія Cricothyrotomy

Транспортування до стаціонару Transportation to hospital

Верхня трахеотомія Upper tracheotomy

80 / 200
На місці пригоди один реаніматолог розпочинає реанімаційні заходи. Яке співвідношення частоти непрямого масажу серця до частоти штучного дихання необхідно обрати в даному випадку? At the scene of the accident, one resuscitator begins resuscitation measures. What ratio of the frequency of indirect heart massage to the frequency of artificial respiration should be chosen in this case?

4:1 4:1

10:2 10:2

30:2 30:2

5:1 5:1

7:1 7:1

81 / 200
У породіллі 16-ти років почалась гіпотонічна кровотеча у ранньому післяпологовому періоді. Об’єм крововтрати 1,6% від маси тіла, Ps- 115/хв., систолічний АТ- 80 мм рт.ст., ЦВТ- 35 мм вод.ст. Який найбільш імовірний діагноз? A 16-year-old woman in labor started hypotonic bleeding in the early postpartum period. Blood loss volume 1.6% of body weight, Ps- 115/min., systolic blood pressure - 80 mm Hg, CVT - 35 mm Hg. What is the most likely diagnosis?

Гіпотонічна кровотеча у ранньому післяпологовому періоді. Геморагічний шок III ступеня Hypotonic bleeding in the early postpartum period. III degree hemorrhagic shock

Гіпотонічна кровотеча у ранньому післяпологовому періоді. Геморагічний шок IV ступеня Hypotonic bleeding in the early postpartum period. IV degree hemorrhagic shock

Гіпотонічна кровотеча у ранньому післяпологовому періоді. Геморагічний шок I ступеня Hypotonic bleeding in the early postpartum period. Hemorrhagic shock of the 1st degree

Гіпотонічна кровотеча у ранньому післяпологовому періоді. Геморагічний шок II ступеня Hypotonic bleeding in the early postpartum period. Hemorrhagic shock of the II degree

Гіпотонічна кровотеча у ранньому післяпологовому періоді без ознак геморагічного шоку Hypotonic bleeding in the early postpartum period without signs of hemorrhagic shock

82 / 200
У хворого 37-ми років, що вживав напередодні копчену рибу власного виробництва, поступово розвинулись м’язова слабкість, неможливість читати газетний текст, на другий день - порушення ковтання. Об’єктивно: стан тяжкий, ціаноз шкіри, страбізм, ЧД-50/хв., Ps- 120/хв. Голос гугнявий, парез зіниць, метеоризм 2 ступеня. Випорожнення відсутні 2 доби. Який лікувальний захід показаний у першу чергу? A 37-year-old patient, who had consumed smoked fish of his own production the day before, gradually developed muscle weakness, the inability to read a newspaper text, and on the second day - a swallowing disorder . Objectively: the condition is severe, cyanosis of the skin, strabismus, BH-50/min., Ps- 120/min. Voice hoarse, paresis of the pupils, flatulence of the 2nd degree. There have been no bowel movements for 2 days. What treatment measure is indicated in the first place?

Штучне дихання ”рот у рот” Mouth-to-mouth artificial respiration

Штучна вентиляція легень Artificial lung ventilation

Введення діурєтиків Introduction of diuretics

Трахеостомiя Tracheostomy

Призначення прозерину Destination of proserin

83 / 200
Хворий 40-ка років скаржиться на інтенсивні болі в горлі при ковтанні, виражену задишку, що посилюється у горизонтальному положенні і супроводжується нападами задухи. При непрямій ларингоскопії виявлений абсцес надгортанника. Відділи гортані розташовані нижче оглянути не вдається. Від трахеотомії хворий категорично відмовився. На висоті чергового нападу задухи розвинулися ціаноз, судоми та наступила зупинка дихання. Яку першочергову допомогу слід надати? A 40-year-old patient complains of intense pain in the throat when swallowing, pronounced shortness of breath, which worsens in a horizontal position and is accompanied by attacks of suffocation. During indirect laryngoscopy, an epiglottis abscess was detected . It is not possible to examine the departments of the larynx located below. The patient categorically refused a tracheotomy. At the height of another attack of suffocation, cyanosis, convulsions and respiratory arrest occurred. What first aid should be provided?

Конікотомія Conicotomy

Інтубація трахеї Tracheal intubation

Штучна вентиляція легенів методом з роту в рот Artificial lung ventilation by the mouth-to-mouth method

Ендоларингеальне розкриття абсцесу під контролем прямої ларингоскопії Endolaryngeal abscess opening under the control of direct laryngoscopy

Трахеотомія Tracheotomy

84 / 200
Хворому 32-х років необхідне переливання еритроцитарної маси. При визначенні групової належності крові пацієнта за системою еритроцитарних антигенів АВ0 аглютинація спостерігалась в краплях з сироватками 0 (I), А (II) та В (III). Кров якої групи потрібно перелити пацієнту? A 32-year-old patient needs an erythrocyte mass transfusion. When determining the patient's blood group according to the erythrocyte antigen system AB0, agglutination was observed in drops with sera 0 (I), A (II) and B (III). Blood of which group should be transfused to the patient?

- -

0 (I ) 0 (I )

А (II) A (II)

В (III) In (III)

АВ (IV) AB (IV)

85 / 200
У пацієнтки в післяпологовому періоді гостро виникло значне зниження настрою, катастрофічне наростання відчуття непереносимої туги, безвихідності, відчаю. Пацієнтка тривожна, не знаходить собі місця, кричить, стогне, плаче, причиняє самоушкодження, активно висловлює суїцидальні наміри. Який найбільш імовірний діагноз? In the postpartum period, the patient experienced a significant drop in mood, a catastrophic increase in the feeling of unbearable longing, hopelessness, despair. The patient is anxious, cannot find a place for herself, screams, moans, cries, causes self-harm, actively expresses suicidal intentions. What is the most likely diagnosis?

Епілептичні сутінки Epileptic Twilight

Травматичний делірій Traumatic delirium

Тяжкий депресивний епізод Severe depressive episode

Гострий приступ шизофренії Acute attack of schizophrenia

Аментивний тип порушення свідомості Amentative type of unconsciousness

86 / 200
У хлопчика 15-ти років раптово підвищилася температура тіла до 39,5oC, виник біль у горлі під час ковтання. На другу добу на обличчі, шиї, верхній частині тулуба на тлі гіперемованої шкіри тулуба виник дрібнокрапковий розеольозний висип з блідим носогубним трикутником. Який найбільш імовірний діагноз? A 15-year-old boy's body temperature suddenly rose to 39.5oC, there was a pain in the throat when swallowing. On the second day, the face, neck, upper part on the trunk, against the hyperemic skin of the trunk, a small-dot roseolosis rash with a pale nasolabial triangle appeared. What is the most likely diagnosis?

Тнфекційний мононуклеоз Infectious mononucleosis

Вітряна віспа Chicken Pox

Краснуха Krasnukha

Кір Measles

Скарлатина Scarlatina

87 / 200
Хворий 44-х років звернувся до дільничного лікаря на 7-й день хвороби зі скаргами на підвищену температуру тіла, поганий сон, закреп. Початок хвороби поступовий. В ході огляду: блідість шкірних покривів, температура - 38,2oC, ЧСС- 68/хв. Печінка збільшена. Які методи дослідження допоможуть підтвердити діагноз? A 44-year-old patient turned to the district doctor on the 7th day of illness with complaints of high body temperature, bad sleep, constipation. The onset of the disease is gradual. During examination: pallor of the skin, temperature - 38.2oC, heart rate - 68/min. The liver is enlarged. What research methods will help confirm the diagnosis?

Полімеразна ланцюгова реакція на віруси гепатитів Polymerase chain reaction for hepatitis viruses

Гемокультура, реакція Відаля Hemoculture, Vidal reaction

Паразитоскопічне дослідження фекалій Parasitoscopic examination of feces

РМАЛ РМАЛ

Реакція Пауль-Буннеля Paul-Bunnel reaction

88 / 200
Жінка 38-ми років впродовж 23-х років страждає на часті, серійні генералізовані судомні напади. Прогресує зниження пам’яті, інертність мислення, запальність. 12 годин тому розпочалися генералізовані тоніко-клонічні напади. Впродовж останніх двох годин між нападами свідомість не поновлюється. Який першочерговий захід надання допомоги? A 38-year-old woman has been suffering from frequent, serial generalized convulsive seizures for 23 years. Progressive memory loss, inertia of thinking, irritability. 12 hours ago generalized tonic-clonic seizures have begun. During the last two hours between seizures, consciousness does not return. What is the first aid measure?

Введення в клізмі хлоралгідрату Chloral hydrate enema

Введення внутрішньом’язово магнію сульфату Intramuscular administration of magnesium sulfate

Введення внутрішньовенно діазепаму Intravenous diazepam

Введення внутрішньовенно натрію оксибутирату Intravenous administration of sodium oxybutyrate

Введення внутрішньом’язово гексеналу Intramuscular injection of hexanal

89 / 200
У хворого 44-х років скарги на загальну слабкість, гіпертермію до 39oC, свербіння шкіри, висип, 6олісність i припухлість суглобів. Об’єктивно: АТ- 70/40 мм рт.ст., папульозномакульозний висип, генералізоване збільшення лімфовузлів, припухлість суглобів, збільшення селезінки. 7 днів тому у травмпункті провели первинну хірургічну обробку рани стопи та ввели протиправцеву сироватку. Вкажіть причину розвитку даного стану: A 44-year-old patient complains of general weakness, hyperthermia up to 39oC, itching of the skin, rash, swelling and swelling of the joints. Objectively: АТ- 70/ 40 mm Hg, papulo-macular rash, generalized enlargement of lymph nodes, swelling of joints, enlargement of the spleen. 7 days ago, primary surgical treatment of the foot wound was performed in the trauma center and anti-tetanus serum was administered. Specify the reason for the development of this condition:

Анафілактична реакція Anaphylactic reaction

Менінгіт Meningitis

Сепсис Sepsis

Геморагічний васкуліт Hemorrhagic vasculitis

Сироваткова хвороба Serum sickness

90 / 200
У потерпілого 36-ти років, що переніс закриту черепно-мозкову травму, в процесі проведення ехо-енцефалоскопії виявлено зміщення серединного еха на 5 мм. Про що це свідчить? In a 36-year-old victim who suffered a closed craniocerebral injury, during the echo-encephaloscopy, a 5 mm shift of the median echo was found. What does this indicate ?

Перелом основи черепа Skull base fracture

Ехо-ЕС патології не виявила Echo-ES did not detect pathology

Наявність підоболонкової гематоми Presence of submucosal hematoma

Тріщина кісток склепіння черепа Crack of the bones of the vault of the skull

Травматичний субарахноїдальний крововилив Traumatic subarachnoid hemorrhage

91 / 200
У 15-річного хлопчика діагностовано бубонну форму чуми. Які антибактеріальні препарати слід призначити дитині? A 15-year-old boy was diagnosed with bubonic plague. What antibacterial drugs should be prescribed to the child?

Гентаміцин або нетроміцин Gentamicin or Netromycin

Стрептоміцин або хлорамфенікол Streptomycin or Chloramphenicol

Пеніцилін або цефотаксим Penicillin or cefotaxime

Бісептол або сульфален Biseptol or sulfalen

Тетрациклін або доксициклін Tetracycline or doxycycline

92 / 200
У дитини 10-ти років на тлі введення гепарину з приводу геморагічного васкуліту різко погіршився стан: АТ-70/30 мм рт.ст., блювота, посилились болі в животі. Запідозрено крововилив у наднирники. Яка тактика лікаря? The condition of a 10-year-old child, against the background of heparin administration due to hemorrhagic vasculitis, worsened sharply: blood pressure 70/30 mm Hg, vomiting, increased pain in the stomach. Hemorrhage in the adrenal glands is suspected. What are the doctor's tactics?

Ввести знеболюючі засоби Enter painkillers

Ввести великі дози метилпреднізолону Inject large doses of methylprednisolone

Почати інфузію допаміну Start dopamine infusion

Ввести протаміну сульфат Enter protamine sulfate

Зменшити дозу гепарину Reduce heparin dose

93 / 200
У дівчинки 3-х тижнів з’явилось блювання після кожного годування. Вага при народженні 3200 г, під час огляду - 3000 г. При огляді підшкірно- жирова клітковина відсутня на животі, тулубі. Еластичність, тургор тканин знижені. Визначається перистальтика шлунка. Стул 'голодний'. Лікарем виставлено попередній діагноз пілоростеноз. Яка подальша тактика? A 3-week-old girl vomited after every feeding. Weight at birth 3200 g, during examination - 3000 g. During examination, subcutaneous fat absent on the abdomen, trunk. Elasticity, turgor of tissues are reduced. Peristalsis of the stomach is determined. The stool is 'hungry'. The doctor made a preliminary diagnosis of pylorostenosis. What are the further tactics?

Призначення метоклопраміду Prescription of metoclopramide

Призначення корекції харчування Appointment of nutritional correction

Госпіталізація у педіатричне відділення Hospitalization in the pediatric department

Госпіталізація у відділення дитячої хірургії Hospitalization in the department of pediatric surgery

Призначення оральної регідратації Prescription of oral rehydration

94 / 200
Під час застілля один з молодих людей, не проковтнувши їжу, почав розповідати історію та раптово поперхнувся. Втратив здатність говорити, спроба відкашлятися ефекту не надала, з’явилося наростаюче утруднення подиху, ціаноз. Постраждалий в свідомості. Який рекомендований обсяг допомоги? During the feast, one of the young people, without swallowing the food, began to tell a story and suddenly coughed. He lost the ability to speak, the attempt to clear his throat had no effect, an increasing difficulty breathing, cyanosis. The victim is conscious. What is the recommended amount of help?

Запрокинути голову, відкрити рота, висунути нижню щелепу Throw back the head, open the mouth, stick out the lower jaw

Нанести удари по спині та здійснити компресію живота Strike the back and perform abdominal compression

Підручними засобами виконати термінову трахеостомію Perform an emergency tracheostomy with improvised means

Відкрити рота та здійснити спробу видалити стороннє тіло Open mouth and attempt to remove foreign body

Провести компресію грудної клітки Perform chest compressions

95 / 200
Новонароджений в стані апное з ціанозом. Після відновлення прохідності дихальних шляхів та проведення стимуляції і вентиляції під позитивним тиском протягом 30 секунд самостійне дихання не з’явилося. На 60 секунді ЧСС становить 45/хв. Подальші лікувальні дії? A newborn in a state of apnea with cyanosis. After restoring the patency of the respiratory tract and carrying out stimulation and ventilation under positive pressure for 30 seconds, independent breathing did not appear. At 60 seconds Heart rate is 45/min. Further treatment actions?

Подальше спостереження за станом Further monitoring of the condition

Розпочати непрямий масаж серця та продовжувати вентиляцію під позитивним тиском Start indirect cardiac massage and continue positive pressure ventilation

Розпочати введення адреналіну Start adrenaline

Розпочати непрямий масаж серця Start indirect heart massage

Продовжувати вентиляцію під позитивним тиском Continue positive pressure ventilation

96 / 200
Хворий 43-х років викликав швидку медичну допомогу. Скаржиться на різкий біль у лівій половині живота з іррадіацією в ліву ногу. В анамнезі сечокам’яна хвороба. Об’єктивно: шкіра бліда, Ps- 100/хв., АТ- 160/90 мм рт.ст. Симптом Пастернацького різко позитивний зліва. Почати лікування найбільш доцільно із призначення: A 43-year-old patient called an ambulance. He complains of sharp pain in the left half of the abdomen with radiation to the left leg. He has a history of urolithiasis. Ob' objectively: the skin is pale, Ps- 100/min., BP- 160/90 mm Hg. Pasternacki's symptom is sharply positive on the left. It is most appropriate to start the treatment with the appointment:

Баралгін внутрішньовенно Baralgin intravenously

Уролесан перорально Urolesan oral

Морфін внутрішньовенно Morphine IV

Анальгін внутрішньом’язово Analgin intramuscularly

Спазмалгон перорально Spazmalgon orally

97 / 200
Хлопчик 8-ми років, катаючись на ковзанах на річці, несподівано провалився під лід. Після вилучення з води: шкіра ціанотична, холодна, не дихає, Ps- 36/хв. Які першочергові заходи необхідно провести? An 8-year-old boy, while skating on the river, unexpectedly fell under the ice. After being removed from the water: the skin is cyanotic, cold, not breathing, Ps- 36 /min. What priority measures should be taken?

Штучне дихання Artificial respiration

Загальне зігрівання General warming

Транспортування до лікарні Transportation to hospital

Вливання до рота гарячого напою Pouring a hot drink into the mouth

Непрямий масаж серця Indirect heart massage

98 / 200
Хворому 22-х років з політравмою в реанімаційній залі проводиться серцево-легеневомозкова реанімація. Який критерій ефективності закритого масажу серця має найбільшу прогностичну цінність? A 22-year-old patient with polytrauma is undergoing cardiopulmonary resuscitation in the intensive care unit. Which criterion for the effectiveness of closed heart massage has the greatest prognostic value?

Аускультативно вислуховуються тони серця Heart tones are auscultated

Відновлення самостійного дихання Restoration of independent breathing

Звуження зіниць Pupillary Constriction

Наявність пульсації на сонній артерії Presence of pulsation on the carotid artery

Наявність пульсу на променевої артерії Presence of a pulse on the radial artery

99 / 200
У новонародженої дитини на третю добу життя з’явилися ознаки шлунково-кишкової кровотечі (блювання з домішками крові, мелена), постгеморагічної анемії. З якого розчину слід розпочати парентеральне лікування геморагічного синдрому у дитини? On the third day of life, a newborn child showed signs of gastrointestinal bleeding (vomiting with blood impurities, melena), posthemorrhagic anemia. From which solution should parenteral treatment of hemorrhagic syndrome in a child?

Амінокапронова кислота Aminocaproic acid

Тромбоцитарна маса Platelet mass

Свіжозаморожена плазма Fresh-frozen plasma

Глюкозо-сольовий розчин Glucose-saline solution

Реополіглюкін Rheopoliglyukin

100 / 200
У роділлі 25-ти років із вродженою вадою серця в I періоді пологів з’явився кашель, задишка, харкотиння, болі за грудиною, тахікардія. АТ- 90/60 мм рт.ст., to- 36,8oC. Роділля займає вимушене сидяче положення. Який найбільш імовірний діагноз? A 25-year-old woman in labor with a congenital heart defect developed cough, shortness of breath, sputum, chest pain, tachycardia in the first period of labor. AT-90/ 60 mm Hg, to- 36.8oC. The woman in labor takes a forced sitting position. What is the most likely diagnosis?

Інфаркт міокарда Myocardial infarction

Розрив матки Rupture of uterus

Напад бронхіальної астми Bronchial asthma attack

Емболія навколоплідними водами Amniotic fluid embolism

Набряк легень Pulmonary edema

101 / 200
У новонародженої доношеної дитини віком 2 дні розвинулося багаторазове блювання з домішками жовчі, наростає здуття живота. Меконіальний стілець відсутній від народження, гази не відходять. Об’єктивно: живіт різко здутий, виражена венозна сітка на передній черевній стінці, контуруються роздуті петлі кишечника, після очисної клізми стул та гази не відходять. Рентгенологічно виявляються розширені та заповнені газом петлі товстої кишки. Який найбільш імовірний діагноз? A 2-day-old newborn full-term child developed multiple vomiting with bilious impurities, abdominal distension is increasing. Meconium stool is absent from birth, gases do not pass. Objectively: abdomen sharply distended, a marked venous network on the anterior abdominal wall, swollen loops of intestines are contoured, stools and gases do not pass after a cleansing enema. X-rays show enlarged and gas-filled loops of the colon. What is the most likely diagnosis?

Виразково-некротичний ентероколіт новонародженого Necrotic ulcerative enterocolitis of a newborn

Вроджена низька непрохідність кишок Congenital low intestinal obstruction

Адрено-генітальний синдром Adreno-genital syndrome

Вроджена висока непрохідність кишок Congenital high intestinal obstruction

Перитоніт новонароджених Neonatal peritonitis

102 / 200
На заводі розірвалась ємність з білим фосфором, в результаті чого 10 працівників одержали опіки. Яку допомогу необхідно надати постраждалим негайно? At the plant, a container with white phosphorus burst, as a result of which 10 workers received burns. What help should be provided to the victims immediately?

Промивання опіків кислотою Acid burn wash

Промивання опіків водою Washing burns with water

Промивання опіків фізрозчином Washing burns with saline

Промивання опіків спиртом Washing burns with alcohol

Промивання опіків розчином мідного купоросу Washing of burns with a solution of copper sulphate

103 / 200
З вогнища радіаційної катастрофи до медичного пункту полку доставлено 15 потерпілих. Через 20-25 хвилин після вибуху потерпілі стали відчувати різку слабкість, запаморочення, нудоту, з’явилось невпинне блювання. Стан хворих тяжкий. Кволі, апатичні, на запитання відповідають з великою затримкою. ЧСС- 120/хв., з частими екстрасистолами, АТ- 70/30 мм рт.ст., ЧД-28/хв. За даними індивідуального дозиметра доза отриманого випромінювання склала 8 Гр. Якій стадії гострої променевої хвороби відповідає наведена симптоматика? 15 victims were brought from the site of the radiation disaster to the regimental medical center. 20-25 minutes after the explosion, the victims began to feel sharp weakness, dizziness, nausea, there was an incessant vomiting. The condition of the patients is serious. Weak, apathetic, they answer questions with a long delay. Heart rate - 120/min., with frequent extrasystoles, BP - 70/30 mm Hg, BP - 28/min. According to the data of the individual dosimeter, the dose received radiation amounted to 8 Gy. What stage of acute radiation sickness corresponds to the given symptomatology?

III

I I

IV IV

V V

II II

104 / 200
Вагітна в терміні 32 тижні страждає на міастенію. З’явились розлади дихання, тахікардія, психомоторне збудження, яке змінюється млявістю, апатією, парезом кишечнику та сфінктерів. Першочергові засоби невідкладної допомоги: A 32-week pregnant woman suffers from myasthenia gravis. Respiratory disorders, tachycardia, psychomotor excitement appeared, which is replaced by lethargy, apathy, paresis of the intestines and sphincters. First-line remedies emergency care:

Призначення прозерину Proserin appointment

Профілактика гіпоксії плоду Prevention of fetal hypoxia

Термінове розродження Urgent delivery

ШВЛ с застосуванням міорелаксантів курареподібної дії Ventilation with curare-like muscle relaxants

Введення транквілізаторів Introduction of tranquilizers

105 / 200
Пацієнтка гінекологічного відділення 32-х років, з загостренням хронічного лівостороннього сальпінгоофоріту під час внутрішньовенного введення цефтріаксону раптово зблідла, посиніла. Лікарем констатована відсутність свідомості, широкі зіниці, відсутність реакції на світло. З чого необхідно починати реанімаційні заходи? A 32-year-old female patient of the gynecology department, with an exacerbation of chronic left-sided salpingo-oophoritis, suddenly turned pale and blue during intravenous administration of ceftriaxone. The doctor stated that she was unconscious, her pupils were wide, there was no reaction into the light. Where should resuscitation measures be started?

Вище місця введення препарату накласти джгут Put a tourniquet above the place of drug injection

Штубація та проведення ШВЛ Intubation and ventilation

Покласти хвору, повернути її голову в бік, висунути нижню шелепу Put the patient down, turn her head to the side, stick out the lower jaw

Місце введення алергену обколоти 0,1% розчином адреналіну The place of introduction of the allergen is lapped with a 0.1% adrenaline solution

Непрямій масаж серця Indirect heart massage

106 / 200
При огляді у новонародженої дитини шкіра ціанотична, неритмічне дихання, ЧСС- 80/хв., зниження м’язового тонусу, виражене ослаблення рефлексів. Перший етап реанімації слід розпочати з: When examining a newborn baby, the skin is cyanotic, irregular breathing, heart rate - 80/min, decreased muscle tone, marked weakening of reflexes. The first stage of resuscitation should be started with:

Штучної вентиляції легень Artificial lung ventilation

Оксигенотерапії Oxygenotherapy

Закритого масажу серця Closed heart massage

Введення розчину адреналіну Injection of adrenaline solution

Відновлення прохідності дихальних шляхів Restoration of airway patency

107 / 200
Дитина 7-ми років госпіталізована з явищами адинамії, що змінюється на збудження, сонливості, задухи, блювання. Об’єктивно: to-40,1oC, Ps- 189/хв., виражена жовтяниця, тремор кінцівок, печінковий запах з рота, зменшення розмірів печінки. Кров: білірубін 200 мкмоль/л. Який найбільш імовірний діагноз? A 7-year-old child was hospitalized with symptoms of adynamia, changing to excitement, drowsiness, suffocation, vomiting. Objectively: to-40.1oC, Ps- 189/min., marked jaundice, limb tremors, liver odor from the mouth, reduction in liver size. Blood: bilirubin 200 μmol/l. What is the most likely diagnosis?

Панкреатична кома Pancreatic coma

Менінгоенцефаліт Meningoencephalitis

Гостра печінкова недостатність Acute liver failure

Ацетонемічний синдром Acetonemic syndrome

Діабетична кома Diabetic coma

108 / 200
Пологи у 36 тижнів. Вагітність перебігала на тлі загрози переривання у 28 тижнів, гострої респіраторної вірусної інфекції у 24 тижні. Навколоплідні води світлі. Новонароджена дівчинка з масою 2400 г, довжиною 45 см. При народженні самостійне дихання відсутнє, м’язовий тонус значно знижений. Після проведення штучної вентиляції легенів за допомогою маски та мішка 100% киснем впродовж 30 секунд самостійне дихання не відновилося, частота серцевих скорочень складає 50/хв., зберігається акроціаноз. Яка подальша тактика лікаря? Delivery at 36 weeks. The pregnancy proceeded against the background of the threat of termination at 28 weeks, an acute respiratory viral infection at 24 weeks. The amniotic fluid is light. A newborn girl weighing 2400 g , 45 cm long. At birth, there is no independent breathing, muscle tone is significantly reduced. After performing artificial ventilation of the lungs using a mask and a bag with 100% oxygen for 30 seconds, independent breathing did not resume, the heart rate is 50/min, acrocyanosis persists What are the doctor's next tactics?

Корекція гіповолемії Correction of hypovolemia

Киснева допомога вільним потоком Free flow oxygen support

Санація верхніх дихальних шляхів Upper respiratory tract rehabilitation

Введення адреналіну Adrenaline injection

Непрямий масаж серця Indirect heart massage

109 / 200
Хворий 38-ми років протягом тривалого часу відчував зубний біль. Згодом з’явилися біль голови і ока, загальна слабкість, підвищилася температура тіла. Об’єктивно: хемоз кон’юнктиви, застійна ін’єкція очного яблука, екзофтальм, обмеження рухів очного яблука. В крові: лейкоцитоз, ШОЕ- 24 мм/год. На рентгенограмі орбіт патології не виявлено. Який найбільш імовірний діагноз? A 38-year-old patient had a toothache for a long time. Later, headache and eye pain appeared, general weakness, body temperature rose. Objectively: chemosis conjunctivae, congestive injection of the eyeball, exophthalmos, restriction of eyeball movements. In the blood: leukocytosis, ESR - 24 mm/h. No pathology was detected on the x-ray of the orbits. What is the most likely diagnosis?

Ретробульбарний крововилив Retrobulbar hemorrhage

Теноніт Tenonite

Целюліт орбіти Cellulitis of the orbit

Запальний псевдотумор орбіти Inflammatory pseudotumor of the orbit

Флегмона орбіти Phlegmon of the orbit

110 / 200
У роділлі 24-х років у пологах раптом погіршилося серцебиття плоду, матка в постійному гіпертонусі, з’явилися значні кров’янисті виділення з піхви. Роділля зблідла, Ps- 130/хв., АТ-80/50 мм рт.ст. Який найбільш імовірний діагноз? In a 24-year-old woman during childbirth, the fetal heartbeat suddenly worsened, the uterus was in constant hypertonicity, significant vaginal discharge appeared. The woman in labor turned pale, Ps - 130/min., BP-80/50 mm Hg. What is the most likely diagnosis?

Клінічно вузький таз Clinically narrow pelvis

Розрив матки Rupture of uterus

Розрив шийки матки Rupture of the cervix

Передчасне відшарування плаценти Premature placental abruption

Гострий апендицит Acute appendicitis

111 / 200
Хворий 51-го року доставлений у відділення кардіореанімації (інтенсивної терапії), скаржиться на тривалий пекучий загруднинний біль, який виник 40 хвилин тому, задишку. Об’єктивно: в легенях - дихання з жорстким відтінком, тони серця різко ослаблені, АТ- 110/70 мм рт.ст. На ЕКГ виявлено підйом сегменту ST на 5 мм над ізолінією у відведеннях I, V1-V4 та депресію сегмента ST у відведеннях III і aVF. Який препарат потрібно застосувати у даній ситуації? A 51-year-old patient was brought to the cardioresuscitation (intensive care) unit, complaining of long-lasting burning chest pain that started 40 minutes ago, shortness of breath. Objectively: in the lungs - breathing with a harsh tone, heart sounds are sharply weakened, BP - 110/70 mm Hg. The ECG revealed an elevation of the ST segment by 5 mm above the isoline in leads I, V1-V4 and depression of the ST segment in leads III and aVF What drug should be used in this situation?

Нітрогліцерин Nitroglycerin

Аміодарон Amiodarone

Альтеплаза Alteplase

Добутамін Dobutamine

Метопролол Metoprolol

112 / 200
Хвора 64-х років перебуває на лікуванні в кардіологічному відділенні з приводу гіпертонічної хвороби. На момент огляду АТ- 200/110 мм рт.ст., ЧСС- 96/хв. Застосування якого препарату в даній ситуації ПРОТИПОКАЗАНО? A 64-year-old patient is being treated in the cardiology department for hypertension. At the time of examination, blood pressure is 200/110 mm Hg, heart rate is 96 /min. The use of which drug is CONTRAINDICATED in this situation?

Рибоксин Riboxin

Каптоприл Captopril

Фуросемід Furosemide

Клофелін Clofelin

Добутамін Dobutamine

113 / 200
Хворий 27-ми років знаходиться на стаціонарному лікуванні з приводу вірусного гепатиту A 27-year-old patient is undergoing inpatient treatment for viral hepatitis

На 7-й день лікування після вечері великою кількістю м’ясної їжі у хворого пропав апетит, з’явились блювання, гикавка, запаморочення. Хворий збуджений. Шкіра жовтяничного кольору, геморагії на шкірі, тремор м’язів, сповільнена мова. Розміри печінки зменшилися. Діагностовано гостру печінкову недостатність. Введення якого препарату, що використовується в лікуванні печінкової недостатності, дозволить зменшити ендогенну інтоксикацію? On the 7th day of treatment, after dinner with a large amount of meat, the patient lost his appetite, vomiting, hiccups, and dizziness appeared. The patient is excited. The skin of the jaundice color, hemorrhages on the skin, muscle tremors, slowed speech. The size of the liver has decreased. Acute liver failure has been diagnosed. The introduction of which drug used in the treatment of liver failure will reduce endogenous intoxication?

Лактулоза 6- Небіволол Lactulose 6- Nebivolol

20 мл 40% глюкози 20 ml of 40% glucose

Орнідазол Ornidazole

100 мл ізотонічного розчину 100 ml of isotonic solution

114 / 200
У дитини 7-ми років внаслідок падіння з турніка виник біль у лівому підребер’ї. При ультразвуковому дослідженні є підозра на розрив селезінки у воріт та кровотечу. Який метод лікування показаний? A 7-year-old child developed pain in the left hypochondrium as a result of falling from a horizontal bar. An ultrasound examination shows a suspicion of rupture of the spleen in the portal vein and bleeding. What method is treatment indicated?

Динамічне спостереження за хворим Dynamic monitoring of the patient

Лапаротомія, перев’язка артерії селезінки Laparotomy, splenic artery ligation

Спленектомія Splenectomy

Переливання еритроцитарної маси Transfusion of erythrocyte mass

Лапароцентез, страхувальний дренаж Laparocentesis, insurance drainage

115 / 200
Пацієнт 50-ти років, що знаходиться у інфарктному відділенні з трансмуральним інфарктом, раптово знепритомнів. Об’єктивно: зупинка серцевої діяльності та дихання, електромеханічна дисоціація. Розпочато серцево-легеневу реанімацію. Які наступні дії? A 50-year-old patient in the infarct department with a transmural infarction suddenly fainted. Objectively: cardiac and respiratory arrest, electromechanical dissociation. Cardiovascular -pulmonary resuscitation. What are the next actions?

Внутрішньовенне введення лідокаїну Intravenous administration of lidocaine

Внутрішньовенне введення строфантину Intravenous administration of strophanthin

Внутрішньовенне введення новокаї-наміду Intravenous administration of novocaine-namide

Дефібриляція Defibrillation

Внутрішньосерцеве введення адреналіну Intracardiac administration of epinephrine

116 / 200
Дівчинка 15-ти років раптово поскаржилась на запаморочення, потемніння в очах, після чого знепритомніла. Тривалий час дитина знаходилась у душному приміщені. Об’єктивно: дівчинка без свідомості, шкірні покриви бліді. Дистальні відділи верхніх та нижніх кінцівок холодні. Відзначаються тахікардія, ниткоподібний пульс, зниження артеріального тиску. Який препарат на догоспітальному етапі треба призначити? A 15-year-old girl suddenly complained of dizziness, darkening of the eyes, after which she fainted. The child was in a stuffy room for a long time. Objectively: the girl is unconscious , the skin is pale. The distal parts of the upper and lower extremities are cold. Tachycardia, a thread-like pulse, and a decrease in blood pressure are noted. What drug should be prescribed at the pre-hospital stage?

Кофеїн бензонат Caffeine Benzonate

Ефедрин Ephedrine

Адреналін Adrenaline

Допамін Dopamine

Мезатон Mesaton

117 / 200
У хворого 72-х років раптово розвинулось блювання кров’ю з крововтратою до 1,0 л. Об’єктивно: АТ- 70/40 мм рт.ст., ЧСС- 112/хв., ЦВТ- 0. Які інфузійні середовища першочергово треба застосувати для лікування? A 72-year-old patient suddenly developed vomiting of blood with blood loss up to 1.0 L. Objectively: blood pressure - 70/40 mm Hg ., heart rate - 112/min., CVT - 0. What infusion media should be used primarily for treatment?

Кристалоїдні розчини Crystalloid solutions

Жирові емульсії Fat emulsions

Свіжозаморожена плазма Fresh-frozen plasma

Колоїдні розчини Colloidal solutions

Розчини глюкози Glucose solutions

118 / 200
Хворий 45-ти років скаржиться на різкий біль в грудному відділі хребта. 3 дні тому отримав травму в ділянці спини при падінні з висоти, за допомогою не звертався. Об’єктивно: загальний стан задовільний, біль під час пальпації в ділянці остистих відростків Th 9-10 хребців. М’язова сила в нижніх кінцівках знижена до 3-4 балів. Колінні та ахілові рефлекси підвищені, симптом Бабінського з двох сторін. Яка патологія у хворого? A 45-year-old patient complains of sharp pain in the thoracic region of the spine. 3 days ago he suffered an injury in the back when falling from a height, he did not seek help. About objectively: the general condition is satisfactory, pain during palpation in the area of ​​the spinous processes of the Th 9-10 vertebrae. Muscle strength in the lower limbs is reduced to 3-4 points. Knee and Achilles reflexes are increased, Babinski's symptom on both sides. What pathology in the patient?

Травматичне пошкодження хребта та спинного мозку Traumatic damage to the spine and spinal cord

Енцефаломієліт Encephalomyelitis

Остеохондроз грудного відділу хребта Osteochondrosis of the thoracic spine

Забій м’яких тканин спини Bruise of the soft tissues of the back

Ниркова коліка Renal colic

119 / 200
Жінку 77-ми років доставлено в лікарню без свідомості. Неврологічний статус: лівобічна параплегія. Протромбін за Квіком становить 108%. На КТ-сканах відзначається: у правій півкулі головного мозку ділянка з нерівномірним зниженням оптичної щільності. Який патологічний стан розвинувся у пацієнтки? A 77-year-old woman was brought to the hospital unconscious. Neurological status: left-sided paraplegia. Quick's prothrombin is 108%. CT scans show: in the right hemisphere an area of ​​the brain with an uneven decrease in optical density. What pathological condition has developed in the patient?

Субарахноїдальний крововилив Subarachnoid hemorrhage

Менінгоенцефаліт Meningoencephalitis

Ішемічний інсульт Ischemic stroke

Метастази у головний мозок Metastases in the brain

Геморагічний інсульт Hemorrhagic stroke

120 / 200
Хворий 58-ми років багато років зловживає алкоголем, на даний час має справжні запої, останній із яких закінчився два дні тому. Скаржиться на безсоння, тривогу, слабкість. Об’єктивно: тремор всього тіла, дезорієнтований, вважає, що він у в’язниці. Бачить навколо величезних пацюків, відчуває, як вони вгризаються йому в ноги, настрій нестійкий: то сміється, то плаче, часто впадає в напівсонний стан. Мова нерозбірлива, переходить в бубоніння. Соматичний стан важкий. Яке лікування є найбільш доцільним? A 58-year-old patient has been abusing alcohol for many years, currently has real binges, the last of which ended two days ago. He complains of insomnia, anxiety, weakness. Objectively: Whole body tremors, disoriented, thinks he is in prison. Sees huge rats around, feels them gnawing at his legs, mood is unstable: sometimes laughs, sometimes cries, often falls into a half-asleep state. Speech is unintelligible , turns into bubbling. The somatic condition is severe. What treatment is most appropriate?

Аміназин + діуретики Aminazine + diuretics

Інтенсивна детоксикація + седуксен Intensive detoxification + seduxen

Антигістамінні + коректори Antihistamines + correctors

Галоперидол + аміназин Haloperidol + aminazine

Препарати вісмуту + антибіотики Bismuth drugs + antibiotics

121 / 200
У вагітної у терміні 38 тижнів, що лежить на спині, раптово з’явилося утруднене дихання, блідість шкірних покривів, холодний липкий піт, знизився АТ. Встановіть діагноз невідкладного стану: A 38-week pregnant woman, lying on her back, suddenly developed difficulty breathing, pale skin, cold sticky sweat, decreased blood pressure. Diagnose an emergency status:

Синдром стиснення нижньої порожнистої вени Inferior vena cava compression syndrome

Пізній гестоз Late gestosis

Емболія навколоплідними водами Amniotic fluid embolism

Відшарування плаценти Detachment of the placenta

Розрив матки Rupture of uterus

122 / 200
Пацієнт 59-ти років проходить курс реабілітаційного лікування після перенесеного інфаркту міокарда. Гемодинамічні показники стабільні. Фракція викиду 50%. Які засоби профілактики раптової серцевої смерті потрібно застосувати у даного пацієнта? A 59-year-old patient is undergoing rehabilitation treatment after a myocardial infarction. Hemodynamic parameters are stable. Ejection fraction is 50%. What means of prevention of sudden cardiac death should be used in this patient the patient?

β-адреноблокатори, іАПФ, статини β-blockers, ACE inhibitors, statins

Діуретики, іАПФ, статини Diuretics, ACE inhibitors, statins

Хірургічна корекція Surgical correction

Аміодарон, α-адреноблокатори Amiodarone, α-blockers

Антагоністи кальцію, іАПФ, стати-ни Calcium antagonists, ACE inhibitors, stop

123 / 200
Хворий поступив у клініку у вкрай важкому стані: кінцівки холодні, акроціаноз, шкіра і слизові сухі, темні кола навколо очей, повторне блювання, живіт безболісний, АТ- 40/0 мм рт.ст., багаторазові водянисті випорожнення без домішок. Діагноз: холера. Вкажіть ступінь зневоднення: The patient was admitted to the clinic in an extremely serious condition: cold extremities, acrocyanosis, dry skin and mucous membranes, dark circles around the eyes, repeated vomiting, pain-free abdomen, blood pressure 40 /0 mm Hg, multiple watery stools without impurities. Diagnosis: cholera. Specify the degree of dehydration:

ІІІ ІІІ

V V

ІІ II

IV IV

І And

124 / 200
Хворого 47-ми років доставлено в токсикологічне відділення в тяжкому стані. Об’єктивно: свідомість за типом оглушення, АТ- 75/40 мм рт.ст., ЧСС- 40/хв. Із анамнезу: 1 годину тому прийняв 3 таблетки клофеліну із суїцидальною метою. Діагноз: отруєння клофеліном. Яка тактика лікаря? A 47-year-old patient was brought to the toxicology department in serious condition. Objectively: consciousness according to the type of stunning, blood pressure - 75/40 mm Hg, Heart rate - 40/min. From the anamnesis: 1 hour ago, he took 3 tablets of clofeline with suicidal intent. Diagnosis: clofeline poisoning. What are the doctor's tactics?

Кордіамін внутрішньовенно, інфузійна терапія Cordiamine intravenously, infusion therapy

Інфузійна терапія, атропіну сульфат внутрішньовенно, промивання шлунка Infusion therapy, intravenous atropine sulfate, gastric lavage

Преднізолон внутрішньовенно, інфузійна терапія, промивання шлунка Prednisolone intravenously, infusion therapy, gastric lavage

Інфузійна терапія, промивання шлунка Infusion therapy, gastric lavage

Промивання шлунка, спостереження Gastric lavage, observation

125 / 200
На 4-ту добу після переливання несумісної по групі крові, у хворого різко зменшився діурез, розвинулася анурія, різко погіршився загальний стан, підвищився артеріальний тиск. При лабораторних дослідженнях: креатинін плазми - 680 мкмоль/л, сечовина плазми - 24 мкмоль/л. Яке захворювання і яку його стадію можна припустити в першу чергу? On the 4th day after the transfusion of incompatible blood group, the patient's diuresis decreased sharply, anuria developed, the general condition deteriorated sharply, blood pressure increased. During laboratory tests : plasma creatinine - 680 μmol/l, plasma urea - 24 μmol/l. What disease and what stage of it can be assumed in the first place?

Гемотрансфузійна гостра ниркова недостатність, анурія Hemotransfusion acute renal failure, anuria

Гемотрансфузійний шок, постренальна гостра ниркова недостатність, анурія Hemotransfusion shock, postrenal acute renal failure, anuria

Гострий інтерстиціальний нефрит, обтураційна анурія Acute interstitial nephritis, obstructive anuria

Постгеморагічна гостра ниркова недостатність, анурія Posthemorrhagic acute renal failure, anuria

Анафілактичний шок, гостра ниркова недостатність, анурія Anaphylactic shock, acute renal failure, anuria

126 / 200
Дитина 11-ти років грала у футбол та під час падіння отримала травму. Встановлено попередній діагноз: закритий вивих правого передпліччя. Який обсяг допомоги повинен надати лікар на місці пригоди? An 11-year-old child was playing football and was injured during a fall. The preliminary diagnosis was established: closed dislocation of the right forearm. What amount of help should the doctor provide at the scene of the accident ?

Восьмиподібна пов’язка на правий ліктьовий суглоб Eight-shaped bandage on the right elbow joint

Знеболювання, транспортна іммобілізація косинкою Anesthesia, transport immobilization with a scarf

Косинкова пов’язка на праву верхню кінцівку Handkerchief bandage on the right upper limb

Знеболювання, спроба вправлення вивиху Anesthesia, attempt to repair dislocation

Знеболювання, транспортна іммобілізація шиною Anesthesia, transport immobilization with a splint

127 / 200
Пішохід 62-х років під час ожеледиці послизнувся та впав на руку на бордюр тротуару. Є локалізований біль у ділянці верхньої третини лівого передпліччя. Спостерігаються тахікардія, підвищення артеріального тиску, припухлість, деформація, синець та крепітація в ділянці верхньої третини лівого передпліччя, рухи кінцівки неможливі. Можна припустити пошкодження верхньої третини лівого передпліччя. Як ви будете транспортувати хворого? A 62-year-old pedestrian slipped and fell on his hand on the curb of the sidewalk during the ice. There is localized pain in the upper third of the left forearm. Tachycardia, increased blood pressure are observed , swelling, deformation, bruising and crepitus in the area of ​​the upper third of the left forearm, movements of the limb are impossible. Damage to the upper third of the left forearm can be assumed. How will you transport the patient?

В напівсидячому положенні з палкою в ліктьових згинах In a semi-sitting position with a stick in the elbows

Прибинтувавши праву верхню кінцівку до грудної клітини та здорової кінцівки Bandaging the right upper limb to the chest and healthy limb

Наклавши шину Крамера з фіксацією плечового, ліктьового, променевозап’ясткового суглобів Having applied Kramer's splint with fixation of shoulder, elbow, radiocarpal joints

Наклавши пов’язку Дезо з валиком в пахвовій ділянці Having applied a Deso bandage with a roller in the armpit

Наклавши шину Крамера від верхньої третини плеча до п’ястно-фалангових суглобів Putting a Kramer splint from the upper third of the shoulder to the carpo-phalangeal joints

128 / 200
У дитини 6-ти років на 2-у добу після гемотрансфузії наявні біль і гіперемія шкіри в області лівого ліктьового згину (місце венепункції), обмеження рухливості в суглобі. При пальпації - болючий інфільтрат у підшкірній клітковині без чітких меж по ходу кубітальної вени протяжністю 4-5 см, симптом флюктуації негативний. Пахвові лімфовузли на стороні ураження не збільшені. Набряку лівої кисті немає. Рухи пальців кисті вільні і безболісні. Який найбільш імовірний діагноз? On the 2nd day after hemotransfusion, a 6-year-old child has pain and hyperemia of the skin in the area of ​​the left elbow bend (venipuncture site), limited mobility in the joint. On palpation - a painful infiltrate in the subcutaneous tissue without clear boundaries along the cubital vein, 4-5 cm long, the symptom of fluctuation is negative. The axillary lymph nodes on the affected side are not enlarged. There is no swelling of the left hand. The movements of the fingers of the hand are free and painless. What is the most likely diagnosis?

Тромбофлебіт кубітальної вени Thrombophlebitis of cubital vein

Бешиха лівої верхньої кінцівки Left upper extremity rash

Абсцес лівого передпліччя Left forearm abscess

Паравазальна флегмона Paravasal phlegmon

Постінфузійний паравазальний інфільтрат Postinfusion paravasal infiltrate

129 / 200
Робітник АЄС змушений тимчасово залишатися на забрудненій радіоактивними речовинами місцевості. Які заходи необхідно йому вжити? A nuclear power plant worker is forced to temporarily stay in an area contaminated with radioactive substances. What measures should he take?

Негайно одягнути респіратор та провітрити приміщення Immediately wear a respirator and ventilate the room

Не виконувати жодних дій до проведення масової евакуації Do not take any action until mass evacuation

Одягнути чоботи, рукавички та комбінезон Put on boots, gloves and overalls

Герметизація приміщення та дезактивація внутрішніх поверхонь Sealing of the room and decontamination of internal surfaces

Самостійно виконати евакуацію Evacuate yourself

130 / 200
У хворої 57-ми років раптово з’явився біль розпираючого характеру в лівій гомілці. Підвищилася температура тіла до 38oC. Захворіла три дні тому. Об’єктивно: шкіра лівої ступні та гомілки помірно ціанотична, блищить. Ліва ступня та гомілка помітно збільшені в об’ємі. Окружність лівої гомілки більша окружності правої на 4 см. Пальпація литкового м’язу болюча. Симптом Хоманса позитивний. Пульсація на дистальних артеріях лівої стопи ослаблена. Який найбільш імовірний діагноз? A 57-year-old patient suddenly developed a throbbing pain in her left lower leg. The body temperature rose to 38oC. She became ill three days ago. Objectively: skin the left foot and lower leg is moderately cyanotic, shiny. The left foot and lower leg are noticeably enlarged. The circumference of the left lower leg is 4 cm larger than the right one. Palpation of the calf muscle is painful. Homans' symptom is positive. Pulsation on the distal arteries of the left foot is weakened. What the most likely diagnosis?

Неврит сідничного нерва Sciatic neuritis

Хвороба Бюргера Burger's disease

Гострий тромбоз вен лівої гомілки Acute venous thrombosis of the left leg

Гострий тромбофлебіт поверхневих вен Acute thrombophlebitis of superficial veins

Гострий артеріальний тромбоз Acute arterial thrombosis

131 / 200
У хворого під час операції на шлунку було ушкоджено селезінкову артерію, в результаті чого виникла масивна кровотеча, яка призвела до зупинки кровообігу. Найдоцільніше проводити: The patient's splenic artery was damaged during a stomach operation, as a result of which massive bleeding occurred, which led to the cessation of blood circulation. It is most expedient to carry out:

Введення гідрокортизону Injection of hydrocortisone

Прямий масаж серця Direct heart massage

Непрямий масаж серця Indirect heart massage

Абдомінальну компресію Abdominal compression

Електрокардіостимуляцію Electronic cardiostimulation

132 / 200
У хворого після автомобільної аварії з’явились скарги на різку задишку, кашель з кров’ю. Об’єктивно: шкіра бліда, ціанотична. Підшкірна емфізема в ділянці грудної клітки, живота, шиї справа. Аускультативно: справа дихання не вислуховується. Ps-130/хв., АТ95 /60 мм рт.ст., Ht- 0,25, Hb-88 г/л. Які першочергові заходи лікаря на догоспітальному етапі? After a car accident, the patient complained of shortness of breath, coughing up blood. Objectively: the skin is pale, cyanotic. Subcutaneous emphysema in the chest area chest, abdomen, neck on the right side. Auscultation: breathing is not heard on the right side. Ps-130/min., AT95 /60 mm Hg, Ht- 0.25, Hb-88 g/l. What are the primary measures of the doctor at the pre-hospital stage ?

Трансфузія еритромаси відповідно до групової і резусної належності Erythromass transfusion according to group and rhesus affiliation

Активна аспірація повітря справа Active air aspiration on the right

Колоїди, еритроцитарна маса, кристалоїди Colloids, erythrocyte mass, crystalloids

Негайна штучна вентиляція легень Immediate CPR

Дофамін 2-5 мкг/кг/хв., інфузія колоїдних розчинів Dopamine 2-5 μg/kg/min, infusion of colloidal solutions

133 / 200
Під час транспортування хворого реанімаційною бригадою в лікарню наступила раптова зупинка серця. За який термін часу людині без будь-яких видимих ознак життя і за відсутності каротидного пульсу необхідно почати серцево-легеневу реанімацію для профілактики мозкового дефіциту? During the transportation of the patient by the resuscitation team to the hospital, a sudden cardiac arrest occurred. In what period of time should a person without any visible signs of life and in the absence of a carotid pulse begin cardiopulmonary resuscitation -pulmonary resuscitation for the prevention of brain deficiency?

5 хвилин 5 minutes

3 хвилини 3 minutes

10 секунд 10 seconds

15 хвилин 15 minutes

60 секунд 60 seconds

134 / 200
На медичному пункті полку хворий 25-ти років зі скаргами на слабкість, нудоту, запаморочення. Біля 3 годин був ядерний вибух. Шкіра бліда, волога, ЧДР- 28/хв., периферичні лімфовузли не збільшені, геморагій немає, дихання везикулярне, тони приглушені; Ps- 106/хв., АТ-110/70 мм рт.ст. Живіт болісний по ходу тонкого кишківника. Набряків немає. Показник індивідуального дозиметра 3,9 Гр. Чим обумовлені клінічні прояви захворювання? At the regimental medical center, a 25-year-old patient complains of weakness, nausea, dizziness. About 3 o'clock there was a nuclear explosion. The skin is pale, moist, ChDR- 28 /min., peripheral lymph nodes are not enlarged, there are no hemorrhages, breathing is vesicular, tones are muffled; Ps- 106/min., BP-110/70 mm Hg. Abdomen is painful along the course of the small intestine. There is no edema. Individual dosimeter reading 3, 9 Gr. What causes the clinical manifestations of the disease?

Гостра токсикоінфекція Acute toxic infection

Гостра променева хвороба I ступеня Acute radiation sickness of the first degree

Гостра променева хвороба IV ступеня Acute radiation sickness of the IV degree

Гостра променева хвороба III ступеня Acute radiation sickness of the III degree

Гостра променева хвороба II ступеня Acute radiation sickness of the II degree

135 / 200
Хворого 28-ми років екстрено доставлено до операційної з приводу профузної шлунковокишкової кровотечі. Загальний стан тяжкий, свідомість - оглушення, АТ- 80/40 мм рт.ст., ЧСС-110/хв., олігурія. Вкажіть об’єм крововтрати: A 28-year-old patient was urgently taken to the operating room due to profuse gastrointestinal bleeding. The general condition is severe, consciousness - stupor, blood pressure - 80/40 mm Hg. , HR-110/min., oliguria. Specify the volume of blood loss:

Крововтрати немає No blood loss

20-25% дефіциту ОЦК (1250-1750 мл ) 20-25% deficit of BCC (1250-1750 ml)

>40% дефіциту ОЦК (>2000 мл ) >40% BCC deficit (>2000 ml)

15% дефіциту ОЦК (750-1250 мл ) 15% deficit of BCC (750-1250 ml)

30-40% дефіциту ОЦК (>1750 мл) 30-40% BCC deficiency (>1750 ml)

136 / 200
При проведенні реанімаційних заходів дитині 5-ти років із зупинкою кровообігу у позалікарняних умовах було інтубовано трахею, але венозний доступ відсутній. Який лікарський засіб може бути введено ендотрахеально? During resuscitation of a 5-year-old child with circulatory arrest outside the hospital, the trachea was intubated, but there was no venous access. What medicine can be administered endotracheally?

Бретиліум Bretylium

Атропін Atropine

Трісамін Trisamine

Хлорид кальцію Calcium chloride

Гідрокарбонат натрію Sodium bicarbonate

137 / 200
Триває значний викид радіонуклідів у атмосферу з пошкодженого ядерного реактора. Доза опромінення всього тіла за перші 10 діб у дорослих становить 500 мЗв. Які заходи захисту треба запровадити? A significant release of radionuclides into the atmosphere from a damaged nuclear reactor continues. The dose of radiation to the whole body for the first 10 days in adults is 500 mSv. What protective measures should be implemented?

За таких умов потрібна евакуація населення із зон радіоактивного забруднення Under such conditions, population evacuation from radioactive contamination zones is required

Обмежити вживання забруднених радіонуклідами харчових продуктів та питної води Limit consumption of food and drinking water contaminated with radionuclides

Потрібно провести йодну профілактику It is necessary to carry out iodine prophylaxis

Ніякі, тому що дози опромінення не перевищують допустимі рівні None, because radiation doses do not exceed permissible levels

Використовуються укриття та засоби захисту органів дихання та шкірного покрову Shelter and means of protection of respiratory organs and skin are used

138 / 200
Очікувана доза опромінення населення, що мешкає на забрудненій радіонуклідами території, за перший рік становить 500 мЗв. Яких заходів захисту потребує це населення? The expected radiation dose of the population living in the territory contaminated with radionuclides for the first year is 500 mSv. What protection measures does this population need?

Потрібно провести йодну профілактику It is necessary to carry out iodine prophylaxis

За даних умов потрібне переселення або евакуація населення Under these conditions, resettlement or evacuation of the population is required

Обмежити вживання забруднених радіонуклідами харчових продуктів та питної води Limit consumption of food and drinking water contaminated with radionuclides

Використовуються укриття та засоби захисту органів дихання та шкірного покриву Shelter and means of protection of respiratory organs and skin are used

Ніяку, тому що дози х-опромінення не перевищують допустимі рівні None, because x-radiation doses do not exceed permissible levels

139 / 200
Солдат доставлений у непритомному стані до медичної роти з казарми. Зі слів товаришів по службі, приблизно за 6 годин до цього він прав обмундирування в закритому приміщенні в розчині технічної рідини. Об’єктивно: непритомний, однак реакція на больові подразники збережена. Шкіра і видимі слизові оболонки бліді, у повітрі, що видихається, солодкий запах. Ps-80/хв., ритмічний, тони серця звучні, АТ- 90/6О мм рт.ст., ЧДР- 18/хв., дихання везикулярне, живіт м’який, неболючий. Який найбільш імовірний діагноз? The soldier was brought in an unconscious state to the medical company from the barracks. According to his comrades, about 6 hours before, he washed his uniform in a closed room in a solution of technical fluid . Objectively: unconscious, but the reaction to painful stimuli is preserved. The skin and visible mucous membranes are pale, there is a sweet smell in the exhaled air. Ps-80/min., rhythmic, heart sounds are audible, BP- 90/6O mm Hg .st., CHDR- 18/min., vesicular breathing, soft, painless abdomen. What is the most likely diagnosis?

Пероральне отруєння хлорованими вуглеводнями важкого ступеня Severe oral chlorinated hydrocarbon poisoning

Отруєння ароматичними вуглеводнями середнього ступеня Moderate aromatic hydrocarbon poisoning

Отруєння етиленгліколем важкого ступеня Severe ethylene glycol poisoning

Інгаляційне отруєння тетраетил-свинцем важкого ступеня Severe tetraethyl lead inhalation poisoning

Інгаляційне отруєння хлорованими вуглеводнями Inhalation poisoning with chlorinated hydrocarbons

140 / 200
Хлопчик у віці 6 місяців хворіє на тяжку форму кашлюка: число нападів кашлю становить 25, напади тяжкі, тривалістю до 10 хвилин, мають 6-8 репризів, закінчуються блюванням. Один з нападів кашлю призвів до апное. Які невідкладні дії? A 6-month-old boy has a severe form of whooping cough: the number of coughing attacks is 25, the attacks are severe, last up to 10 minutes, have 6-8 repetitions, end with vomiting . One of the coughing fits led to apnea. What are the immediate actions?

Лобелін внутрішньом’язово Lobelin intramuscularly

Штучне дихання (в ручному режимі) Artificial respiration (in manual mode)

Налагодити подачу кисню Set up oxygen supply

Преднізолон внутрішньом’язово Prednisone intramuscularly

Седуксен внутрішньом’язово Seduxen intramuscularly

141 / 200
На фоні гіпертермії та сильного головного болю в дитини 5-ти років виникли зорові галюцинації жахливого змісту, марення, дезорієнтація. Шкіра гіперемована, з дрібною висипкою, рожевого кольору. Шкіра носогубного трикутника бліда, ознаки напруження м’язів потилиці, симптоми Керніга, Брудзинського, пожвавлення рефлексів без різниці між сторонами. Який діагноз можна припустити? Against the background of hyperthermia and a severe headache, a 5-year-old child developed visual hallucinations of terrible content, delirium, disorientation. The skin is hyperemic, with a small rash, pink in color. The skin of the nasolabial triangle is pale, there are signs of tension in the muscles of the back of the head, symptoms of Kernig, Brudzinsky, invigoration of reflexes without difference between the sides. What diagnosis can be assumed?

Менінгококовий менінгіт з деліріо-зним синдромом Meningococcal meningitis with delirious syndrome

Деліріозний синдром при алергічному захворюванні Delirious syndrome in allergic disease

Деліріозний синдром при черепно-мозковій травмі Delirious syndrome with brain injury

Деліріозний синдром при отруєнні їжею Delirious syndrome with food poisoning

Коревий менінгіт із деліріозним синдромом Measles meningitis with delirious syndrome

142 / 200
До інфекційного відділення надійшов хворий 20-ти років з ознаками дегідратації ІІІ ступеня. Спостерігаються судоми, блювання фонтаном, різке зниження артеріального тиску, тахікардія, температура тіла - 35,5oC. З якою швидкістю треба проводити внутрішньовенну інфузію для відновлення водно-електролітного балансу? A 20-year-old patient was admitted to the infectious disease department with signs of dehydration of the III degree. There are convulsions, vomiting with a fountain, a sharp drop in blood pressure, tachycardia, body temperature - 35, 5oC. At what speed should an intravenous infusion be carried out to restore the water-electrolyte balance?

50-80 мл/хв 50-80 ml/min

100-120 мл/хв 100-120 ml/min

80-100 мл/хв 80-100 ml/min

30-50 мл/хв 30-50 ml/min

Швидкість не має значення Speed ​​doesn't matter

143 / 200
У чоловіка 28-ми років на фоні бронхопневмонії розвинулася кома. Об’єктивно: ЧСС122 /хв., екстрасистолія. АТ- 80/45 мм рт.ст. Шкіра суха, тургор знижений. Дихання глибоке, шумне, рідке. Різкий запах ацетону. Печінка +5 см. Глікемія - 32 ммоль/л; рН крові - 7,0. Який розчин буде найбільш ефективним для нормалізації дихання? A 28-year-old man developed a coma against the background of bronchopneumonia. Objectively: heart rate 122/min, extrasystole. Blood pressure - 80/45 mmHg. The skin is dry, turgor is reduced. Breathing is deep, noisy, thin. Sharp smell of acetone. Liver +5 cm. Glycemia - 32 mmol/l; blood pH - 7.0. Which solution will be most effective for normalizing breathing?

4,2% розчин натрію бікарбонату 4.2% sodium bicarbonate solution

Рєополіглюкін Rheopoliglyukin

1% розчин калію хлориду 1% potassium chloride solution

5% розчин глюкози 5% glucose solution

0,9% розчин натрію хлориду 0.9% sodium chloride solution

144 / 200
Жінка 25-ти років була доставлена у гінекологічне відділення, вагітність 19 тижнів. Скарги на болі внизу живота, переймоподібного характеру, через деякий час після їх початку з’явились кров’янисті виділення з піхви. При огляді розміри матки відповідають терміну вагітності, цервікальний канал дещо відкритий, шийка матки сформована, розм’якшена. Який найбільш імовірний діагноз? A 25-year-old woman was brought to the gynecology department, 19 weeks pregnant. Complaints of pain in the lower abdomen, of a cramp-like nature, appeared some time after their onset bloody discharge from the vagina. On examination, the size of the uterus corresponds to the term of pregnancy, the cervical canal is slightly open, the cervix is ​​formed, softened. What is the most likely diagnosis?

Завмерла вагітність Frozen pregnancy

Загрозливий аборт Threatened abortion

Позаматкова вагітність Ectopic pregnancy

Аборт в ходу Abort in progress

Неповний аборт Incomplete abortion

145 / 200
Після використання супротивником хімічної зброї у більшості військових спостерігаються кашель, утруднення дихання, слинотеча, які майже зникли після виходу постраждалих з зони хімічного зараження. Через 6 годин у всіх отруєних з’явився різкий кашель з великою кількістю пінистого харкотиння, вислуховуються численні вологі хрипи різного характеру. Яка бойова хімічна речовина була використана супротивником? After the use of chemical weapons by the enemy, the majority of soldiers have cough, difficulty breathing, and salivation, which almost disappeared after the victims left the area of ​​chemical contamination. After 6 hours, all poisoned a sharp cough appeared with a lot of foamy sputum, numerous wet wheezes of various nature are heard. What chemical warfare agent was used by the enemy?

Зарін Zarin

Люїзит Lewisite

Фосген Phosgene

Табун Herd

Адамсит Adamsite

146 / 200
Хворий 83-х років надійшов до приймально-діагностичного відділення з попереднім діагнозом: защемлена пупкова кила. Під час обстеження раптово зблід, втратив свідомість. Дихання відсутнє. Пульсація на магістральних артерій припинилася 30 секунд тому. Зіниці розширені, на світло не реагують. З чого треба розпочати реанімаційні заходи? An 83-year-old patient came to the reception and diagnostic department with a preliminary diagnosis: pinched umbilical hernia. During the examination, he suddenly turned pale, lost consciousness. There is no breathing. Pulsation on the main arteries stopped 30 seconds ago. The pupils are dilated, they do not react to light. What should resuscitation measures be started?

Електродефібриляція Electrodefibrillation

Медикаментозна терапія Drug therapy

Прекардіальний удар Precardiac attack

Закритий масаж серця Closed heart massage

Штучне дихання Artificial respiration

147 / 200
У хворого з черепно-мозковою травмою, який знаходиться в комі, на ШВЛ через ендотрахеальну трубку, при аускультації виявлені ділянки різко ослабленого дихання, та ділянки 'німої легені'. Яка лікувально-діагностична процедура показана? In a patient with a brain injury who is in a coma, on mechanical ventilation through an endotracheal tube, during auscultation areas of sharply weakened breathing and areas of a 'silent lung' were detected What medical and diagnostic procedure is indicated?

Призначити ацетилцистеїн Prescribe acetylcysteine

Механічна стимуляція кашлю Mechanical stimulation of cough

Ввести кофеїн Enter Caffeine

Санаційна фібробронхоскопія Remedial fibrobronchoscopy

Вібромасаж грудної клітки Chest vibration massage

148 / 200
Чоловік 75-ти років поскаржився на інтенсивний біль в нижній половині живота, слабкість, відсутність сечовипускання протягом доби. В анамнезі помірна артеріальна гіпертензія, цукровий діабет, доброякісна гіперплазія простати. Об’єктивно: шкіра бліда, волога; тони серця глухі, ритм правильний ЧСС- 100/хв., АТ- І80/90 мм рт.ст., перкуторно сечовий міхур визначається на 10 см над лоном. Які першочергові заходи слід провести для покращення стану хворого? A 75-year-old man complained of intense pain in the lower half of the abdomen, weakness, no urination during the day. He has a history of moderate arterial hypertension, diabetes, benign prostatic hyperplasia . Objectively: the skin is pale, moist; the heart sounds are dull, the rhythm is correct HR - 100/min., BP - I80/90 mm Hg, percussion the bladder is determined 10 cm above the womb. What primary measures should be taken for improvement of the patient's condition?

Призначення а-адреноблокаторів Prescription of α-adrenoblockers

Внутрішньовенне введення дротаверину Intravenous administration of drotaverine

Термінове хірургічне втручання Urgent surgery

Катетеризація сечового міхура за допомогою катетеру Фолея Catheterization of the urinary bladder using a Foley catheter

Внутрішньовенне введення еналаприлату для швидкого зниження АТ Intravenous administration of enalaprilat for rapid BP reduction

149 / 200
Хвора на варикозну хворобу нижніх кінцівок впала. Виникла сильна кровотеча із варикозно розширеного вузла в середній третині лівої гомілки. Як зупинити кровотечу на догоспітальному етапі? A patient with varicose veins of the lower extremities fell. There was severe bleeding from a varicose node in the middle third of the left lower leg. How to stop the bleeding at the pre-hospital stage?

Накласти джгут на місце ушкодження Apply a tourniquet to the injury

Накласти джгут нижче місця ушкодження Apply a tourniquet below the injury

Накласти стискаючу пов’язку на місце ушкодження Put a compression bandage on the injury

Накласти стискаючу пов’язку вище місця ушкодження Put a compression bandage above the injury

Накласти джгут вище місця ушкодження Apply a tourniquet above the injury

150 / 200
Лікар швидкої допомоги при огляді хворого 19-ти років встановив, що пацієнт захворів раптово: підвищилась температура до 39,2oC, виражений головний біль, часте блювання, що не приносить полегшення. На шкірі сідниць, стегнах, гомілках, тулубі з’явились петехіальний висип, який місцями зливається. Менінгеальні симптоми позитивні. Лікар запідозрив генералізовану форму менінгококової інфекції. Яке лікування необхідно призначити на догоспітальному етапі? During an examination of a 19-year-old patient, the emergency physician found that the patient fell ill suddenly: the temperature rose to 39.2oC, severe headache, frequent vomiting, which brings relief. A petechial rash appeared on the skin of the buttocks, thighs, lower legs, trunk, which merges in places. The meningeal symptoms are positive. The doctor suspected a generalized form of meningococcal infection. What treatment should be prescribed at the pre-hospital stage?

Доксициклін, кортикостероїди Doxycycline, corticosteroids

Роваміцин, кортикостероїди Rovamycin, corticosteroids

Пеніцилін, кортикостероїди Penicillin, corticosteroids

Фізіологічний розчин, контрикал довенно Physiological solution, contractile for life

Левоміцетин, кортикостероїди, інфузійна терапія Levomycetin, corticosteroids, infusion therapy

151 / 200
Породілля 23-х років, на другий день після пологів відчула наростаючий біль в правій нозі, важкість. Об’єктивно: стан хворої середньої важкості. Ps- 100/хв., температура 37,3oC, тони серця приглушені, ритм синусовий, в легенях дихання везикулярне, 18-20/хв. Ліва нога на всьому протязі набрякла, синюшна, порівняно з правою температура знижена. Пульсація артерій лівої ноги на всьому протязі пальпується, ослаблена. Який найбільш імовірний діагноз? A 23-year-old woman in labor, on the second day after giving birth, felt increasing pain in her right leg, heaviness. Objectively: the condition of the patient is of medium severity. Ps- 100/ min., temperature 37.3oC, heart sounds are muffled, sinus rhythm, vesicular breathing in the lungs, 18-20/min. The left leg is swollen throughout, bluish, compared to the right, the temperature is lower. Pulsation of the arteries of the left leg is palpable throughout, weakened. What is the most likely diagnosis?

Тромбоемболія клубової артерії Thromboembolism of the iliac artery

Гострий клубово-стегновий флебо-тромбоз Acute iliac-femoral phlebo-thrombosis

Тромбофлебіт глибоких вен правої нижньої кінцівки, післяпологовий сепсис Thrombophlebitis of the deep veins of the right lower limb, postpartum sepsis

Тромбоемболія стегнової артерії Thromboembolism of the femoral artery

Гострий тромбофлебіт вен правої нижньої кінцівки Acute thrombophlebitis of the veins of the right lower limb

152 / 200
При проведенні закритого масажу серця людини 76-ти років, що знаходиться у стані клінічної смерті, необхідно здійснювати компресії з частотою: When performing a closed heart massage of a 76-year-old person who is in a state of clinical death, it is necessary to perform compressions with the frequency:

Не менше 80 на 1 хвилину At least 80 for 1 minute

Не менше 120 на 1 хвилину At least 120 for 1 minute

Не менше 60 на 1 хвилину At least 60 for 1 minute

Не менше 40 на 1 хвилину At least 40 for 1 minute

Не менше 100 на 1 хвилину At least 100 for 1 minute

153 / 200
В амбулаторію сімейної медицини родичами доставлена хвора 80 років, яка впала з ліжка, відчула різкий біль в ділянці верхньої третини стегна. Стан важкий. Хвора виснажена, погано розуміє питання, не завжди відповідає по суті. Пульс аритмічний, тони серця глухі, миготлива аритмія. Гомілки пастозні, трофічні порушення на шкірі. Нога на боці ушкодження значно ротована назовні. У ділянці великого вертлюга видно синець. Запідозрений черезвертлюговий перелом стегнової кістки. Які анатомічні межі для транспортної іммобілізації слід обрати? An 80-year-old patient was brought to the family medicine outpatient clinic by her relatives, who fell out of bed, felt a sharp pain in the area of ​​the upper third of the thigh. The condition is serious. The patient is exhausted, does not understand the questions well , does not always correspond to the essence. The pulse is arrhythmic, the heart sounds are dull, pulsatile arrhythmia. The lower legs are pasty, trophic disorders on the skin. The leg on the side of the injury is significantly rotated outward. A bruise is visible in the area of ​​the greater acetabulum. Suspected transtrochanteric fracture of the femur. What are the anatomical limits for transport immobilization should be chosen?

Від колінного суглоба до гребеня клубової кістки From knee joint to iliac crest

Від основи пальців до кульшового суглоба From the base of the fingers to the hip joint

Від колінного суглоба до пахвинної ямки From the knee joint to the inguinal fossa

Від основи пальців до пахвинної ямки From the base of the fingers to the inguinal fossa

Від основи пальців до гребеня клубової кістки From the base of the fingers to the crest of the iliac bone

154 / 200
У хворого 47-ми років після нервово-психічного перенапруження з’явився виражений головний біль ('ніби хтось вдарив палкою по голові'), світлобоязнь, загальна гіперестезія. При огляді: анталгічна поза, різко виражена ригідність потиличних м’язів, симптоми Керніга та Брудзинського. За даними рентгенівської комп’ютерної томографії - без патології. Який наступний метод дослідження є найінформативнішим для визначення подальшої тактики ведення пацієнта? A 47-year-old patient developed severe headache ('as if someone had hit his head with a stick'), photophobia, and general hyperesthesia after neuropsychological overstrain. During the examination: antalgic posture, pronounced stiffness of the occipital muscles, Kernig and Brudzinsky symptoms. According to X-ray computed tomography - without pathology. Which next research method is the most informative for determining the further tactics of patient management?

Транскраніальна доплерографія Transcranial Doppler

Аналіз спинномозкової рідини Analysis of cerebrospinal fluid

Огляд очного дна Fundus examination

Магнітнорезонансна томографія Magnetic resonance imaging

Коагулограма Coagulogram

155 / 200
Потерпілого 78-ми років знайдено у дворі будинку тепло вдягненим, лежачим на снігу. Без свідомості. Пульс - 100/хв., ритмічний, АТ-160/100 мм рт.ст., частота дихальних рухів -20/хв. Температура тіла у прямій кишці 35,8oC. Кисті і стопи холодні, звичайного кольору. Пасивні рухи у суглобах без обмежень. Який з діагнозів найбільш вірний? The 78-year-old victim was found in the yard of the house, warmly dressed, lying on the snow. Unconscious. Pulse - 100/min., rhythmic, AT-160/100 mm Hg, frequency of respiratory movements -20/min. Body temperature in the rectum is 35.8oC. Hands and feet are cold, normal color. Passive movements in the joints without restrictions. Which of the diagnoses is the most correct?

Закрита черепно-мозкова травма Closed brain injury

Алкогольне сп’яніння Alcohol intoxication

Загальне охолодження середнього ступеня важкості General cooling of medium severity

Кома невідомого генезу Coma of unknown origin

Відмороження кистей і стоп, дореактивний період Defrosting of hands and feet, pre-reactive period

156 / 200
Лікаря швидкої допомоги викликали до дитини в віці 1 рік, що раптово посиніла та стала задихатися. Лікар застав дитину серед розсипаних дрібних предметів (ґудзики, квасоля і так далі). Стан дитини мало відрізняється від звичайного, продовжує грати. Ознак дихальної недостатності немає. Які дії лікаря? The emergency doctor was called to a 1-year-old child who suddenly turned blue and began to suffocate. The doctor found the child among scattered small objects (buttons, beans, etc.) . The child's condition is little different from usual, he continues to play. There are no signs of respiratory failure. What are the actions of the doctor?

Постукати по спині і стимулювати кашель Tap on the back and stimulate coughing

Оглянути дитині ротову порожнину, не дивлячись на його опір Examine the child's oral cavity, regardless of his resistance

Заспокоїти мати і сказати, що дитині нічого не загрожує і такі напади у дітей бувають Reassure the mother and say that the child is not in danger and such attacks happen to children

Залишити під спостереженням батьків Leave under parental supervision

Провести аускультацію і перкусію органів грудної порожнини та доставити дитину в стаціонар Perform auscultation and percussion of the chest cavity and deliver the child to the hospital

157 / 200
У хворого 15-ти років скарги на відчуття серцебиття, слабкість, які виникли раптово 40 хвилин тому, такі напади відбувались раніше і закінчувалися раптово за декілька хвилин. Викликана бригада швидкої допомоги, при огляді ЧСС до 220/хв., АТ- 115/68 мм рт.ст., стан стабільний. Яке обстеження має бути виконано терміново? A 15-year-old patient complains of heart palpitations, weakness, which appeared suddenly 40 minutes ago, such attacks occurred earlier and ended suddenly in a few minutes. The team was called ambulance, during the examination the heart rate is up to 220/min., blood pressure is 115/68 mm Hg, the condition is stable. What examination should be performed urgently?

Доплер ЕхоКГ Doppler EchoKG

ЕКГ ECG

Проби Ашнера і Вальсальви Aschner and Valsalva tests

Кліно-ортостатична проба Wedge-orthostatic test

Холтеровське моніторування Holter monitoring

158 / 200
Вагітна 19-ти років, в терміні 30 тижнів, поскаржилася на головний біль, порушення зору, блювання з домішками крові. Стан важкий, АТ-160/100 мм рт.ст., склери жовтушні, підшкірні крововиливи, набряки нижніх кінцівок, сеча геморагічного кольору. В крові: тромбоцитопенія. Який найбільш імовірний діагноз? A 19-year-old pregnant woman, at 30 weeks, complained of a headache, visual disturbances, vomiting with blood impurities. The condition is severe, BP-160/100 mm Hg, sclerae are yellowish, subcutaneous hemorrhages, swelling of the lower extremities, hemorrhagic urine. In the blood: thrombocytopenia. What is the most likely diagnosis?

HELLP-синдром HELLP syndrome

Гепатоз вагітних Hepatosis of pregnant women

Еклампсія Eclampsia

ДВЗ -синдром DVZ syndrome

Хвороба Боткіна Botkin's disease

159 / 200
Лікар швидкої допомоги, приїхавши на місце ДТП, діагностував закритий перелом правого стегна в середній третині. Яким повинен бути обсяг допомоги? The ambulance doctor, having arrived at the scene of the accident, diagnosed a closed fracture of the right hip in the middle third. What should be the amount of assistance?

Знеболення введенням в місце перелому 1% р-ну новокаїну, іммобілізація шиною Дітеріхса, внутрішньовенне введення розчинів Anesthesia by injecting 1% novocaine solution into the fracture site, immobilization with Dieterichs bandage, intravenous injection of solutions

Іммобілізація трьома шинами Крамера до кульшового суглоба Immobilization with three Kramer splints to the hip joint

Іммобілізація трьома шинами Крамера до пахвинної ділянки Immobilization with three Kramer splints to the groin

Знеболення, іммобілізація шиною Дітеріхса, інфузійна терапія Anesthesia, immobilization with Dieterichs splint, infusion therapy

Знеболення наркотичними препаратами, фіксація пошкодженої кінцівки до здорової Anesthesia with narcotic drugs, fixation of a damaged limb to a healthy one

160 / 200
До лікаря на прийом звернувся хворий, у якого два дні назад з’явились приступи інтенсивного болю в лівій половині грудної клітки, загальна слабість, лихоманка і біль голови. Сьогодні вранці помітив висип на шкірі грудей. При огляді хворого зліва по ходу 4-5 міжреберного нерва на еритематозному, набряклому фоні є множинні згруповані міхурці із напруженою покришкою, наповнені прозорим вмістом, величиною 2-4 мм в діаметрі. Для якого захворювання характерна вказана симптоматика? A patient came to the doctor for an appointment, who two days ago had attacks of intense pain in the left half of the chest, general weakness, fever and headache. Today in the morning, he noticed a rash on the skin of the chest. When examining the patient on the left side along the course of the 4-5 intercostal nerve, on an erythematous, swollen background, there are multiple grouped vesicles with a tight lid, filled with transparent contents, 2-4 mm in diameter. For which disease is the indicated symptomatology characteristic?

Простий герпес Herpes simplex

Герпетиформний дерматоз Дюринга Dühring's herpetiform dermatosis

Оперізуючий герпес Herpes zoster

Стрептококове імпетиго Streptococcal impetigo

Рожевий лишай Pink lichen

161 / 200
Хворий 38-ми років масою 76 кг з поширеним перитонітом внаслідок гострого деструктивного апендициту був прооперований через 2 доби після початку захворювання. Проведена апендектомія. В палаті інтенсивної терапії через 40 хвилин після операції внутрішньовенно введено 2 г меропенему. Перед введенням проведена алергологічна проба: негативна. Через 40 хвилин після введення антибіотика АТ знизився до 70/40 мм рт.ст., Ps- 120/хв. Діурез - 11 мл/год, ЦВТ- 5 мм вод.ст., гематокрит - 50 г/л, температура тіла - 39oC, шкірні покриви гіперемовані, тахіпное до 40 за хвилину, ціаноз слизових оболонок. Який стан розвинувся у хворого? A 38-year-old patient weighing 76 kg with widespread peritonitis due to acute destructive appendicitis was operated on 2 days after the onset of the disease. An appendectomy was performed. In the intensive care unit after 40 minutes after the operation, 2 g of meropenem was administered intravenously. Before the administration, an allergy test was performed: negative. 40 minutes after the administration of the antibiotic, blood pressure decreased to 70/40 mm Hg, Ps- 120/min. Diuresis - 11 ml/h, CVT- 5 mm H2O, hematocrit - 50 g/l, body temperature - 39oC, hyperemic skin, tachypnea up to 40 per minute, cyanosis of mucous membranes. What condition has developed in the patient?

Геморагічний шок Hemorrhagic shock

Гіповолемічний шок Hypovolemic shock

Інфекційно-токсичний шок Infectious-toxic shock

Кардіогенний шок Cardiogenic shock

Анафілактичний шок Anaphylactic shock

162 / 200
Жінка 26-ти років надійшла до пологового стаціонару в строку пологів для проведення планового кесарського розтину з приводу рівномірно звуженого тазу. Під час інтубації трахеї виникли технічні труднощі, що зумовили розвиток стану клінічної смерті. Одним з етапів реанімаційних заходів є: A 26-year-old woman came to the maternity hospital during the delivery period for a planned cesarean section due to a uniformly narrowed pelvis. During tracheal intubation, technical difficulties arose, which led to development of the state of clinical death. One of the stages of resuscitation measures is:

Надати пацієнтці положення Тренделенбурга Give the patient the Trendelenburg position

Проведення вставленої абдомінальної компресії Performing inserted abdominal compression

Негайно підняти нижні кінцівки пацієнтки Immediately raise the patient's lower limbs

Надати пацієнтці положення Фовлера Give patient Fowler's position

Екстрене розродження шляхом проведення кесарського розтину Emergency delivery by caesarean section

163 / 200
Солдат 19 років під час пожежі в БТР отримав циркулярні футлярні опіки нижніх кінцівок IV ступеню на площі 30% поверхні тіла. Яке лікування постраждалого слід провести під час надання кваліфікованої хірургічної допомоги? During a fire in an armored personnel carrier, a 19-year-old soldier received IV-degree circular sheath burns of the lower extremities on an area of ​​30% of the body surface. What treatment should be given to the victim during the provision of qualified surgical help?

Протишокова терапія Antishock therapy

Протишокова терапія та пересадка шкіри Antishock therapy and skin grafting

Висічення некрозу та пересадка шкіри Necrosis excision and skin grafting

Протишокова терапія та некротомії вражених ділянок Anti-shock therapy and necrotomies of affected areas

Протишокова терапія та накладання асептичних пов’язок Anti-shock therapy and application of aseptic bandages

164 / 200
У новонародженої дівчинки після народження відсутнє серцебиття (асистолія). Шляхи введення адреналіну при первинній реанімації новонароджених: A newborn girl has no heartbeat (asystole) after birth. Ways of introducing adrenaline during primary resuscitation of newborns:

Внутрішньом’язовий Intramuscular

Внутрішньошкірний Intradermal

Внутрішньом’язовий та внутрішньовенний Intramuscular and intravenous

Внутрішньом’язовий та інтратрахеальний Intramuscular and intratracheal

Внутрішньовенний та інтратрахеальний Intravenous and intratracheal

165 / 200
Хворий 34-х років скаржиться на біль та скутість у грудному та поперековому відділах хребта після довготривалого сидіння та в нічний час. Об’єктивно: невелика сутулість, ходіння скутне, позитивні симптоми Томаєра і Шобера, ознаки правобічного іридоцикліту. Рентгенографія кісток тазу: крижово-здухвинні з’єднання нерівні, нечіткі, ділянки остеосклерозу, їх щілини з обох боків нерівномірно звужені. Який попередній діагноз? A 34-year-old patient complains of pain and stiffness in the thoracic and lumbar regions of the spine after sitting for a long time and at night. Objectively: slight stooping, stiff gait , positive Tomayer and Schober symptoms, signs of right-sided iridocyclitis. X-ray of pelvic bones: sacro-joint joints are uneven, indistinct, areas of osteosclerosis, their slits on both sides are unevenly narrowed. What is the previous diagnosis?

Анкілозуючий спондилоартрит Ankylosing spondylitis

Остеоартроз із ураженням хребта Osteoarthrosis with damage to the spine

Хвороба Рейтера, сакроілеїт Reiter's disease, sacroiliitis

Туберкульозне ураження хребта Tuberculosis of the spine

Псоріатичний артрит, сакроілеїт Psoriatic arthritis, sacroiliitis

166 / 200
У потерпілих в результаті аварії на нафтопроводі в перші години спостерігаються сльозотеча, збудження, ейфорія, головний біль, нудота, блювання, атаксія, порушення мови та ковтання. Яка тактика невідкладної терапії є найбільш обґрунтованою? Victims of an oil pipeline accident in the first hours have lacrimation, excitement, euphoria, headache, nausea, vomiting, ataxia, impaired speech and swallowing. What tactics emergency therapy is the most reasonable?

Форсований діурез, куприніл, унітіол Forced diuresis, cuprinil, unitiol

Промивання шлунку, ентеросорбент, аллоксим Gastric lavage, enterosorbent, alloxim

Промивання шлунку, введення вазелінового масла Gastric lavage, introduction of petroleum jelly

Промивання шлунку, унітіол, гемодіаліз Gastric lavage, unitiol, hemodialysis

ШВЛ, оксигенотерапія, кортикостероїди, ліпін Ventilation, oxygen therapy, corticosteroids, lipin

167 / 200
До приймального відділення госпіталю МНС потрапили з зони катастрофи 20 осіб. Під час огляду постраждалого 30-ти років, якого вилучили з-під уламків споруди, визначається: блідість шкірного покриву, у свідомості, деформація правої гомілки, багаточисельні забиття м’яких тканин нижніх кінцівок. До якої групи постраждалих треба призначити цього постраждалого при проведенні медичного сортування? 20 people from the disaster area were admitted to the reception department of the Ministry of Emergency Situations hospital. During the examination of the 30-year-old victim, who was removed from the debris of the building, it was determined: paleness of the skin integument, conscious, deformation of the right lower leg, multiple bruises of the soft tissues of the lower extremities. To which group of victims should this victim be assigned during medical triage?

Потребує симптоматичного лікування Requires symptomatic treatment

Потребує евакуації до іншого лікувального закладу Requires evacuation to another medical facility

Потребує невідкладної медичної допомоги Requires urgent medical care

Допомогу може бути відкладено Help may be delayed

Не потребує медичної допомоги на даний момент Does not require medical attention at this time

168 / 200
У хворого 18-ти років через 3 тижні після перенесеної скарлатини з’явилися набряки на гомілках та повіках, біль у поперековій ділянці, олігурія, дизурія, сеча кольору 'м’ясних помиїв', підвищення температури до 38,5oC. У сечі: реакція лужна, відносна щільність 1028, білок - 0,8 г/л, еритроцити - все поле зору, лейкоцити - 2-4 в п/зору, циліндри гіалінові - 4-6 в п/зору, зернисті - 1-3 в п/зору. Добова екскреція білка - 1 г. Який дiагноз є найбільш імовірним? An 18-year-old patient developed swelling on the lower legs and eyelids, pain in the lumbar region, oliguria, dysuria, colored urine 3 weeks after suffering from scarlet fever. meat scum', an increase in temperature up to 38.5 o C. In urine: alkaline reaction, relative density 1028, protein - 0.8 g/l, erythrocytes - the entire field of vision, leukocytes - 2-4 per vision, hyaline cylinders - 4-6 per eye, granular - 1-3 per eye. Daily excretion of protein - 1 g. What is the most likely diagnosis?

Гострий гломерулонефрит із нефритичним синдромом Acute glomerulonephritis with nephritic syndrome

Тубулоінтерстиціальний нефрит Tubulointerstitial nephritis

Гострий пієлонефрит Acute pyelonephritis

Сечокам’яна хвороба, ниркова колька Urolithiasis, renal colic

Хронічний гломерулонефрит із нефротичним синдромом Chronic glomerulonephritis with nephrotic syndrome

169 / 200
Постраждалий вдома вчинив суїцидальну спробу через повішення. Витягнутий із петлі через 2 хвилини. Свідомість відсутня, проте є виражене локомоторне збудження, періодично виникають короткочасні судоми. Проте рухи збережені в усіх кінцівках. Дихання часте, інспіраторна ядуха. Ціаноз виражений помірно. Тахікардія до 110/хв. АТ170/100 мм рт.ст. Що потрібно зробити в першу чергу? The victim made a suicide attempt at home by hanging. He was pulled out of the noose after 2 minutes. Consciousness is absent, but there is pronounced locomotor excitement, short-term convulsions occur periodically. However, movements are preserved in in all limbs. Breathing is frequent, inspiratory dyspnea. Cyanosis is moderately expressed. Tachycardia up to 110/min. BP170/100 mm Hg. What should be done first of all?

Негайно почати масаж серця Start heart massage immediately

Негайно знизити артеріальний тиск Lower blood pressure immediately

Ввести еуфілін, дати кисень Enter euphilin, give oxygen

Ввести дихальні аналептики (кордіамін) Enter respiratory analeptics (cordiamine)

Ввести сибазон, проводити ШВЛ Enter sibazone, carry out ventilation

170 / 200
Хворий 83-х років, що перебуває в палаті інтенсивної терапії з приводу загострення ішемічної хвороби серця, раптом втратив свідомість. Зафіксовано клінічну смерть. Вкажіть цільові значення сатурації киснем артеріальної крові (SpO2) постраждалого після повернення спонтанного кровообігу: An 83-year-old patient in the intensive care unit due to an exacerbation of coronary heart disease suddenly lost consciousness. Clinical death was recorded. Specify the target values ​​of arterial oxygen saturation blood (SpO2) of the victim after the return of spontaneous blood circulation:

96-98% 96-98%

86-90% 86-90%

85% 85%

91-93% 91-93%

100% 100%

171 / 200
Пацієнт 36-ти років скаржиться на неможливість самостійного сечовипускання при наявності покликів. 3 години тому - падіння в каналізаційний люк з забоєм промежини. Об’єктивно: виділення крові з зовнішнього отвору сечівника, помірно виражена гематома промежини. Ознаків переломів немає. Яка подальша лікувальна тактика? A 36-year-old patient complains of the impossibility of urinating on his own in the presence of urges. 3 hours ago - a fall into a sewer manhole with a cut in the perineum. Objectively: blood discharge from of the external opening of the urethra, a moderately expressed hematoma of the perineum. There are no signs of fractures. What are the further treatment tactics?

Стимуляція діурезу Diuresis stimulation

Відкрите накладення цистостоми Open cystostomy overlay

Встановлення постійного катетера Indwelling catheter

Пункційна цистостомія Puncture cystostomy

Однократна катетеризація One time catheterization

172 / 200
Хворий 76-ти років звернувся до лікаря зі скаргами на наявність безболісних нерухливих утворень в пахвинних ділянках, розмірами до 2-3 см в діаметрі. З анамнезу: останнім часом погіршився апетит, втратив вагу, у випорожненнях періодично відмічає свіжу кров. Яке дослідження для встановлення діагнозу необхідно провести першочергово? A 76-year-old patient turned to the doctor with complaints about the presence of painless, immobile formations in the groin, up to 2-3 cm in diameter. From the history: recently his appetite has worsened, he has lost weight, he periodically notices fresh blood in his stools. What research should be carried out first to establish a diagnosis?

Ультразвукове дослідження лімфовузлів пахвинної ділянки Ultrasound examination of lymph nodes in the groin area

Комп’ютерна томографія Computed tomography

Пальцеве дослідження прямої кишки Digital rectal examination

Ультразвукове дослідження пахвинної ділянки Ultrasound examination of the groin area

Іригографія Irrigography

173 / 200
У хворого періодично виникають напади клонічних судом у лівій руці, які іноді розповсюджуються на ліву половину обличчя. Напад триває 5 хвилин, після чого відмічається слабкість у лівій руці, яка поступово зникає впродовж 2-3 годин. Який варіант судомного синдрому наведено? The patient periodically has attacks of clonic convulsions in the left hand, which sometimes spread to the left half of the face. The attack lasts 5 minutes, after which there is weakness in the left hand, which gradually disappears within 2-3 hours. What variant of the convulsive syndrome is given?

Напад скроневої епілепсії Attack of temporal lobe epilepsy

Напад Джексоновської епілепсії Attack of Jacksonian epilepsy

Малий епілептичний напад Minor seizure

Напад Кожевніковської епілепсії Attack of Kozhevnikov's epilepsy

Генералізований епілептичний напад Generalized epileptic seizure

174 / 200
Під час обстрілу в зоні бойових дій, у солдата з’явились наступні симптоми: став тривожним, надмірно метушливим, гіперактивним, не виконував накази командира, неадекватно реагував на навколишніх. Впродовж двох годин стан покращився, симптоми стали менш вираженими, пам’ять на події, що відбувались, частково втрачена. Поставте імовірний діагноз: During the shelling in the combat zone, the soldier developed the following symptoms: he became anxious, excessively fussy, hyperactive, did not follow the commander's orders, reacted inappropriately to others . Within two hours, the condition improved, the symptoms became less pronounced, the memory of the events that took place was partially lost. Make a probable diagnosis:

Розлад адаптації Adaptation disorder

Гостра реакція на стрес Acute reaction to stress

Гострий поліморфний психотичний розлад Acute polymorphic psychotic disorder

Контузія Contusion

Посттравматичний стресовий розлад Post-traumatic stress disorder

175 / 200
Пацієнт 42-х років, оглянутий лікарем-психіатром, перебуває в стані оглушення, що періодично змінюється тривогою, відчаєм, афективними спалахами, гіперактивністю. На запитання не реагує. Відомо, що стан виник через декілька хвилин після бойових дій та триває 2 години. Який імовірний діагноз? A 42-year-old patient examined by a psychiatrist is in a state of stupor, which periodically alternates with anxiety, despair, affective outbursts, hyperactivity. He does not respond to questions. It is known that the condition arose a few minutes after the hostilities and lasted for 2 hours. What is the probable diagnosis?

Параноїдна реакція Paranoid reaction

Тривожна реакщя Anxious reaction

Іпохондричний невроз Hypochondriac neurosis

Гостра реакція на стрес Acute reaction to stress

Конверсійна реакція Conversion response

176 / 200
Вагітна з прееклампгією важкого ступеню знаходиться у відділенні інтенсивної терапії. Під час обходу поскаржилась на відчуття тяжкості при диханні, загальну слабкість, АТ-135/80 мм рт.ст. та 130/80 мм рт.ст., Ps- 62/хв., частота дихання - 20/хв., знижені сухожильні рефлекси. За останні 12 годин діурез - 200 мл. Впродовж 2-х діб отримує сульфат магнію. Яке ускладнення виникло у пацієнтки? A pregnant woman with severe preeclampsia is in the intensive care unit. During the visit, she complained of a feeling of heaviness when breathing, general weakness, BP-135/80 mm Hg . and 130/80 mm Hg, Ps - 62/min, respiratory rate - 20/min, decreased tendon reflexes. In the last 12 hours, urine output - 200 ml. For 2 days, he receives magnesium sulfate. What is the complication occurred in the patient?

Тромбоемболія легеневої артерії Thromboembolism of the pulmonary artery

Передозування сульфату магнію Magnesium sulfate overdose

Госпітальна пневмонія Nosocomial pneumonia

Транзиторна ішемічна атака Transient ischemic attack

Передвісники еклампсії Precursors of eclampsia

177 / 200
Населення, яке проживає близько атомної електростанції, у зв’язку з аварією на одному з енергоблоків, зазнало радіаційного опромінення. Більшість уражених отримали дозу зовнішнього опромінення 2 Гр. Який вид медичної допомоги необхідно надати ураженим? The population living near a nuclear power plant was exposed to radiation due to an accident at one of the power units. Most of those affected received an external radiation dose of 2 Gy. What what type of medical care should be provided to the injured?

Перша лікарська First medical

Спеціалізована Specialized

Кваліфікована Qualified

Само-взаємодопомога Self-help

Перша долікарська First pre-medical school

178 / 200
На хімічному комбінаті 7 чоловік отримали отруєння невідомою речовиною. Доставлені до токсикологічного відділення в коматозному стані. Об’єктивно: мідріаз, зіниці на світло не реагують, арефлексія, повторне блювання, мимовільне сечовипускання, судоми, ціаноз, to- 35oC, ЧД- 8-10/хв., дихання глибоке, АТ- 80/40 мм рт.ст. Яку антидотну терапію слід застосувати? At the chemical plant, 7 people were poisoned by an unknown substance. They were taken to the toxicology department in a comatose state. Objectively: mydriasis, pupils do not react to light, areflexia, repeated vomiting, involuntary urination, convulsions, cyanosis, to- 35oC, BH- 8-10/min., deep breathing, BP- 80/40 mm Hg. What antidote therapy should be used?

Тетацин-кальцію внутрішньовенно Thetacin-calcium IV

Унітіол внутрішньом’язово Unithiol intramuscularly

Розчин хромосмону внутрішньовенно Chromosmon solution intravenously

5% розчин етанолу внутрішньовенно 5% ethanol solution intravenously

Тіосульфат натрію внутрішньовенно Sodium thiosulfate intravenously

179 / 200
Хворий 30-ти років госпіталізований без свідомості. Зі слів родичів отримує інсулін пролонгованої дії 28 ОД вранці та 14 ОД ввечері. Раптово втратив свідомість. Судоми. Шкіра волога, тонус м’язів підвищений, очні яблука без змін. Ps- 96/хв., АТ- І20/75 мм рт.ст. Язик вологий. Менінгеальні симптоми відсутні. Яка терапія необхідна в першу чергу? A 30-year-old patient is hospitalized unconscious. According to relatives, he receives long-acting insulin 28 units in the morning and 14 units in the evening. He suddenly lost consciousness. Convulsions. The skin is moist, the muscle tone is increased, the eyeballs are unchanged. Ps- 96/min., BP- I20/75 mm Hg. The tongue is wet. There are no meningeal symptoms. What therapy is needed in the first place?

Введення адреналіну підшкірно Injection of epinephrine subcutaneously

Введення гідрокортизону внутрішньовенно Introduction of hydrocortisone intravenously

Введення інсуліну короткої дії внутрішньовенно Introduction of short-acting insulin intravenously

Введення 4% гідрокарбонату натрію внутрішньовенно Introduction of 4% sodium bicarbonate intravenously

Введення 40% розчину глюкози внутрішньовенно Introduction of 40% glucose solution intravenously

180 / 200
Пацієнту 65-ти років, який хворіє облітеруючим атеросклерозом судин нижніх кінцівок, виконали операцію - ампутації ноги на рівні н/3 стегна. В зв’язку з вираженим больовим синдромом в післяопераційному періоді хворому тричі вводився омнопон. Які ускладнення можуть виникнуть у хворого в зв’язку з використанням великих доз наркотичних анальгетиків? A 65-year-old patient suffering from obliterating atherosclerosis of the vessels of the lower extremities underwent an operation - amputation of the leg at the level of the n/3 thigh. Due to severe pain syndrome in the postoperative period, the patient was injected with omnopon three times. What complications may occur in the patient in connection with the use of large doses of narcotic analgesics?

Фібриляція шлуночків серця Ventricular fibrillation

Зупинка дихання Stop breathing

Гостра ниркова недостатність Acute renal failure

Зупинка серця Cardiac arrest

Гостра печінкова недостатність Acute liver failure

181 / 200
Хворий 28-ми років доставлений в лікарню у зв’язку із втратою свідомості. Скаржиться на виражену загальну слабкість, запаморочення. Об’єктивно: загальний стан тяжкий, шкірні покриви бліді. Ps- 110/хв., АТ- 90/60 мм рт.ст. Тони серця ритмічні. В легенях дихання везикулярне. Живіт м’який, нерізко виражена болючість в епігастрії. В яке відділення повинен бути госпіталізований хворий? A 28-year-old patient was taken to the hospital due to loss of consciousness. He complains of pronounced general weakness, dizziness. Objectively: the general condition is severe, skin the integuments are pale. Ps- 110/min., BP- 90/60 mmHg. Heart sounds are rhythmic. Breathing is vesicular in the lungs. Abdomen is soft, mild pain in the epigastrium. In which department should the patient be hospitalized?

У гастроентерологічне відділення To the gastroenterology department

У кардіологічне відділення To the cardiology department

У терапевтичне відділення To the therapeutic department

У хірургічне відділення To the surgical department

У неврологічне відділення To the neurology department

182 / 200
Хворий 68-ми років страждає на сечокам’яну хворобу, конкременти обох нирок. Впродовж останніх 3-х діб на тлі двобічної ниркової кольки відмітив зменшення кількості сечі до 50 мл на добу. За останні 48 годин креатинін крові з нормального рівня збільшився майже вдвічі. Який найбільш імовірний діагноз у даному випадку? A 68-year-old patient suffers from urolithiasis, calculi in both kidneys. During the last 3 days, against the background of bilateral renal colic, he noticed a decrease in the amount of urine to 50 ml per day. In the last 48 hours, the blood creatinine has almost doubled from the normal level. What is the most likely diagnosis in this case?

Двобічний блок нирок Bilateral kidney block

Преренальне гостре ураження нирок Prerenal acute kidney injury

Ренальне гостре ураження нирок Renal acute kidney injury

Постренальне гостре ураження нирок Post-traumatic acute kidney injury

Термінальна стадія хронічного ураження нирок на тлі сечокам’яної хвороби Terminal stage of chronic kidney damage on the background of urolithiasis

183 / 200
До лікарні звернулася родина (двоє дітей та двоє дорослих), які 2 години тому вживали самостійно зібрані гриби. Усі скаржаться на нудоту, біль у животі, слиновиділення. Лікар діагностував отруєння грибами. Якою повинна бути першочергова допомога? A family (two children and two adults) came to the hospital, who 2 hours ago consumed mushrooms they had collected themselves. Everyone complains of nausea, abdominal pain, salivation. The doctor diagnosed mushroom poisoning. What should be the first aid?

Гемодіаліз Hemodialysis

Промивання шлунка Gastric lavage

Форсований діурез Forced diuresis

Парентеральне водне навантаження Parenteral water load

Ентеральне водне навантаження Enteral water load

184 / 200
Хворий побитий невідомими особами, побої наносились руками по голові. Хворий втрачав свідомість на 510 хвилин, мали місце нудота та блювання, загальна слабкість в неврологічному статусі розсіяна мікровогнищева неврологічна симптоматика. Місцеві зміни: забої м’яких тканин голови. Який метод обстеження, найбільш інформативний, слід провести для постановки діагнозу? The patient was beaten by unknown persons, the beatings were inflicted with hands on the head. The patient lost consciousness for 510 minutes, nausea and vomiting occurred, general weakness in the neurological status, scattered microfocal neurological symptoms Local changes: contusions of the soft tissues of the head. What method of examination, the most informative, should be carried out to make a diagnosis?

Рентгенографія черепа X-ray skull

Комп’ютерна томографія головного мозку Computed tomography of the brain

Електроенцефалографія Electroencephalography

Ангіографія судин головного мозку Angiography of cerebral vessels

Ехоенцефалографія Echoencephalography

185 / 200
У хлопчика 5-ти років через 8-10 хвилин після укусу бджоли з’явилися занепокоєння, гіперемія і свербіж шкіри обличчя, нудота, захриплість голосу, свистяче дихання, сплутаність свідомості, блідість пітливість. Який найбільш імовірний діагноз? In a 5-year-old boy, 8-10 minutes after a bee bite, anxiety, hyperemia and itching of the skin of the face appeared, nausea, hoarseness of voice, wheezing, confusion, pallor, sweating. What is the most likely diagnosis?

Інфекційно-токсичний шок Infectious-toxic shock

Анафілактичний шок Anaphylactic shock

Набряк Квінке Quincke edema

Кропивниця Hives

Гострий стенозуючий ларингіт Acute stenosing laryngitis

186 / 200
До постраждалого 38-ми років викликана бригада невідкладної медичної допомоги. Зі слів очевидців хворий травму отримав внаслідок ураження електричним струмом. Об’єктивно: свідомість відсутня, дихання та пульс не визначаються, на ЕКГ - ознаки фібриляції міокарду. Першочерговий захід у даному випадку: The emergency medical team was called to the 38-year-old victim. According to eyewitnesses, the patient was injured due to electric shock. Objectively: consciousness is absent, breathing and pulse are not determined, on the ECG there are signs of myocardial fibrillation. The primary measure in this case:

Введення серцевих глікозидів Introduction of cardiac glycosides

Непрямий масаж серця Indirect heart massage

Інфузійна терапія Infusion therapy

Дефібриляція Defibrillation

Транспортування постраждалого в лікувальний заклад Transportation of the victim to a medical facility

187 / 200
Хворий 28-ми років, з Анголи, госпіталізований в лікарню із скаргами на сильний кашель та задуху, проноси, кровохаркання. Протягом року відмічав пережну гарячку, значну пітливість з профузними нічними потами, різку загальну слабкість. Місяць тому появився тривалий пронос, у калі є домішки слизу і крові. Об’єктивно: стан тяжкий, температура тіла - 40oC, кахексія. На лівому оці виявлена пухлина на рогівці. Шкіра та слизові оболонки бліді. Який діагноз найбільш імовірний у цьому випадку? A 28-year-old patient from Angola was admitted to the hospital with complaints of severe cough and suffocation, diarrhea, hemoptysis. During the year, he noted chronic fever, significant sweating with profuse night sweats, sharp general weakness. A month ago, prolonged diarrhea appeared, there are impurities of mucus and blood in the feces. Objectively: the condition is severe, body temperature - 40oC, cachexia. A tumor on the cornea was found in the left eye. The skin and mucous membranes are pale. What is the most likely diagnosis in this case?

СНІД, генералізована форма AIDS, generalized form

ВІЛ-інфекція HIV infection

Криптоспоридіоз Cryptosporidiosis

Сепсис Sepsis

Пре-СНІД Pre-AIDS

188 / 200
Під час оперативного втручання, що виконувалося в умовах ендотрахеального наркозу, у жінки 82-х років на кардіомоніторі зафіксовано фібриляцію шлуночків. Розпочато реанімаційні заходи. Оберіть шлях введення лікарських засобів під час проведення серцево-легеневої та церебральної реанімації: During the surgical intervention, which was performed under endotracheal anesthesia, an 82-year-old woman had ventricular fibrillation recorded on the cardiomonitor. Resuscitation measures were initiated. Select the route of drug administration during cardiopulmonary and cerebral resuscitation:

Інтраперитонеальний Intraperitoneal

Внутрішньосерцевий Intracardiac

Внутрішньовенний Intravenous

Введення лікарських засобів протипоказано Administration of medicinal products is contraindicated

Внутрішньокістковий Intraosseous

189 / 200
Перша медична допомога (самота взаємодопомога) при радіаційних ураженнях передбачає усунення або послаблення початкових ознак променевої хвороби. З цією метою особовий склад Збройних Сил безпосередньо після вибуху для профілактики первинної реакції бере з індивідуальної аптечки (одну таблетку): First medical aid (alone mutual aid) for radiation injuries involves eliminating or weakening the initial signs of radiation sickness. For this purpose, the personnel of the Armed Forces immediately after the explosion to prevent the initial reaction takes from an individual first-aid kit (one tablet):

Вібрацин Vibracin

Цистамін Cystamine

Будаксим Budaksim

Доксицикліну гідрохлорид Doxycycline hydrochloride

Етаперазин Etaperazin

190 / 200
Пацієнт 75-ти років з дiагнозом: гострий поширений передній інфаркт міокарда, раптово відчув біль у грудній клітці, задишку, що швидко збільшувались. Аускультативно: вологі хрипи над легенями з обох боків. ЧД- 22/хв., АТ- 80/40 мм рт.ст. На ЕКГ: ЧСС-200/хв., комплекси QRS більше 0,12 ',' A 75-year-old patient with a diagnosis of: acute widespread anterior myocardial infarction, suddenly experienced chest pain, shortness of breath, which rapidly increased. Auscultation: moist rales over the lungs on both sides. BH - 22/min., BP - 80/40 mm Hg. On ECG: heart rate - 200/min., QRS complexes more than 0.12 ','

Що необхідно призначити при наданні невідкладної допомоги? What should be prescribed when providing emergency care?

Аміодарон Amiodarone

Кардіоверсія Cardioversion

Адреналін Adrenaline

Лідокаїн 6- Атропін Lidocaine 6- Atropine

191 / 200
З метою вправлення вивиху пальця лівої руки хворому 20-ти років було виконано блокаду променевого нерва. Через 10 хвилин після введення 5 мл 2% розчину лідокаїну хворий почав скаржитись на різку слабкість, порушення зору, нудоту, нестачу повітря, відчуття стискання у грудній клітці. Об’єктивно: шкіра бліда, волога, осиплість голосу, ЧД- 35/хв., ЧСС- 120/хв., АТ- 50/20 мм рт.ст. Який лікарський засіб необхідно ввести негайно? A radial nerve block was performed on a 20-year-old patient for the purpose of correcting a dislocation of a finger on the left hand. 10 minutes after the administration of 5 ml of a 2% lidocaine solution, the patient began to complain of sharp weakness, impaired vision, nausea, lack of air, feeling of tightness in the chest. Objectively: pale skin, moist, hoarseness of voice, heart rate - 35/min., heart rate - 120/min., BP - 50/20 mm Hg .art. What medicine should be administered immediately?

Адреналін Adrenaline

Фуросемід Furosemide

Димедрол Diphenhydramine

Еуфілін Euphilin

Седуксен Seduxen

192 / 200
Хлопчик 16-ти років скаржиться на затримку сечі та різкий біль в надлобковій ділянці. Відомо, що дитина лікувалась в нефрологічному відділенні з приводу гіперурікемії та уролітіазу. Під час обстеження пальпується напружений сечовий міхур в надлобковій ділянці. Викликано ШМД. Яка має бути тактика ведення цього пацієнта? A 16-year-old boy complains of urinary retention and sharp pain in the suprapubic area. It is known that the child was treated in the nephrology department for hyperuricemia and urolithiasis. During the examination a tense bladder is palpated in the suprapubic area. SMD is caused. What should be the management tactics of this patient?

Парацетамол внутрішньо Paracetamol internally

50% розчин анальгіну внутрішньом’-язово 50% analgin solution intramuscularly

0,5% розчин седуксену внутрі-шньом’язово 0.5% seduxen solution intramuscularly

2% розчин но-шпи внутрішньом’я-зово 2% no-spi solution intramuscularly

Фенобарбітал внутрішньо Phenobarbital internally

193 / 200
Хворий 77-ми років звернувся в хірургічну клініку зі скаргами на тупий, ниючий біль в животі з іррадіацією в поперек, нестійкі випорожнення, зниження апетиту, схуднення. Об’єктивно в мезогастріі пальпується пульсуюче утворення з систолічним шумом над ним. Який діагноз найбільш імовірний? A 77-year-old patient came to the surgical clinic with complaints of dull, aching pain in the abdomen radiating to the lower back, unstable bowel movements, decreased appetite, weight loss. Ob' Objectively, a pulsating mass with a systolic murmur above it is palpated in the mesogastric area. What is the most likely diagnosis?

Аневризма черевної аорти Abdominal aortic aneurysm

Синдром Леріша Lerisch Syndrome

Абдомінальний ішемічний синдром Abdominal ischemic syndrome

Рак підшлункової залози Pancreatic cancer

Цироз печінки Liver cirrhosis

194 / 200
У військовослужбовця в зоні бойових дій має місце вивих великого пальця лівої руки. До якої сортувальної категорії з її візуальним позначенням буде віднесений потерпілий? A military serviceman in a combat zone has a sprain of his left thumb. To which sorting category with its visual designation will the victim be assigned?

IV (темно-фіолетовий колір) IV (dark purple color)

II (жовтий колір) II (yellow color)

V (синій колір) V (blue color)

I (червоний колір) I (red color)

III (зелений колір) III (green color)

195 / 200
У вагітної в 32 тижні після падіння при об’єктивному обстеженні лікарем швидкої допомоги виявлено: матка в гіпертонусі, шкіра бліда, АТ- 90/60 мм рт.ст., Ps- 105/хв., частота дихання -22/хв. Вагітна занепокоєна і вказує на локальний біль біля пупка. В якому положенні слід проводити інфузійну терапію під час транспортування до стаціонару? In a pregnant woman at 32 weeks after a fall, an objective examination by an emergency physician revealed: the uterus is hypertonic, the skin is pale, blood pressure - 90/60 mm Hg ., Ps- 105/min., respiratory rate -22/min. The pregnant woman is worried and indicates local pain near the navel. In what position should infusion therapy be carried out during transportation to the hospital?

З опущеним головним кінцем With leading end omitted

На спині з припіднятими ногами On the back with raised legs

З припіднятим головним кінцем With raised head end

Положення не має значення The position does not matter

Положення на лівому боці з припіднятими ногами Position on the left side with raised legs

196 / 200
Хвора 56-ти років, скаржиться на слабкість, біль в м’язах, парестезії в ділянці обличчя, нижніх кінцівок, симетричні судоми, які виникають спонтанно, утруднення дихання, блювання. В анамнезі - два тижні тому перенесла тотальну резекцію щитоподібної залози з приводу дифузного токсичного зобу III ступеня і отримує тироксин 100 мкг на добу. Об’єктивно: свідомість затьмарена, шкіра суха. У верхніх кінцівках - судоми м’язів за типом '”руки акушера', порушений акт ковтання, задишка. Ps- 62/хв., ритмічний. АТ-115/60 мм рт.ст. Позитивні симптоми Хвостека, Труссо. Лабораторно: рівень кальцію в крові -1,3 ммоль/л; глікемія - 5,3 ммоль/л. Який найбільш імовірний діагноз? A 56-year-old patient complains of weakness, muscle pain, paresthesias in the face, lower limbs, symmetrical convulsions that occur spontaneously, difficulty breathing , vomiting. In the anamnesis - two weeks ago she underwent a total resection of the thyroid gland due to a diffuse toxic goiter of the III degree and receives thyroxine 100 mcg per day. Objectively: consciousness is clouded, skin is dry. In the upper extremities - muscle spasms of the type 'hands of an obstetrician', impaired act of swallowing, shortness of breath. Ps- 62/min., rhythmic. BP-115/60 mm Hg. Positive symptoms of Khvostek, Trousseau. Laboratory: calcium level in blood -1.3 mmol/l; glycemia - 5.3 mmol/l. What is the most likely diagnosis?

Епілептичний статус Status Epileptic

Правець Tetanus

Істерія Hysteria

Інсулома Insuloma

Гіпокальціємічна криза Hypocalcemic crisis

197 / 200
У відділення реанімації бригадою швидкої допомоги доставлена дитина 3-х років у непритомному стані. Під час проведення штучного дихання та непрямого масажу серця на ЕКГ реєструється шлуночкова брадикардія 50/хв. Якому із наведених медикаментозних заходів необхідно надати перевагу? An unconscious 3-year-old child was brought to the intensive care unit by the ambulance team. During artificial respiration and indirect heart massage, ventricular bradycardia 50/min was recorded on the ECG Which of the following medical measures should be preferred?

Електрична дефібриляція серця + введення адреналіну Electric heart defibrillation + adrenaline injection

Внутрішньовенне ведення атропіну Intravenous administration of atropine

Внутрішньовенне ведення гідрокарбонату натрію Intravenous administration of sodium bicarbonate

Електрична дефібриляція серця Electrical defibrillation of the heart

Внутрішньовенне введення адреналіну Intravenous administration of epinephrine

198 / 200
Після бою солдат зі зниженою руховою активністю, постійно згадує бій, момент, коли впритул застрелив ворога. Вночі спав погано, постійно просинався від жахів в яких він бачив бойові дії, обличчя вбитого ним. Визначте вид посттравматичної стресової реакції, яка розвинулась у солдата? After the battle, a soldier with reduced motor activity constantly remembers the battle, the moment when he shot the enemy at close range. He slept badly at night, constantly waking up from the horrors in which he saw combat actions , the face of the person killed by him. Determine the type of post-traumatic stress reaction that developed in the soldier?

Галюциноз Hallucinosis

Гіперреактивність Hyperreactivity

Інтрузія Intrusion

Депресія Depression

Уникання Avoid

199 / 200
Військовий знаходиться в секторі укриття на полі бою. Його поранено в живіт. Стогне. На передній черевній стінці обширна рана з випавшими петлями кишківника. Пульс слабкий. Температура повітря +7oC. Які дії? The soldier is in the shelter sector on the battlefield. He has been shot in the stomach. He is moaning. There is an extensive wound on the front abdominal wall with fallen loops of intestines. The pulse is weak. Air temperature + 7oC. What actions?

Вкутати в простирадло та транспортувати до пункту надання допомоги Wrap in a sheet and transport to the aid point

Випавші нутрощі вправити та фіксувати пов’язкою до черевної стінки. Накласти пов’язку на рану живота Fix the fallen viscera with a bandage to the abdominal wall. Put a bandage on the abdominal wound

Вправити нутрощі, що випали, знеболити, транспортувати до пункту надання допомоги Exercise spilled entrails, anesthetize, transport to aid station

Накласти пов’язку на рану живота. Знеболити Put a bandage on the abdominal wound. Anesthetize

Накласти вологу пов’язку на рану живота, випавші нутрощі не вправляти Put a wet bandage on the wound of the abdomen, do not exercise the spilled entrails

200 / 200
В медичну роту доставлено військовослужбовця з вогнепальним пораненням верхньої третини правої гомілки з пошкодженням магістральних судин. Систолічний АТ- 100 мм рт.ст., Ps- 90/хв. При наданні домедичної допомоги годину тому накладено джгут, пов’язку на рану та уведений знеболюючий засіб. В яку чергу необхідно евакуювати пораненого на подальший етап медичної евакуації? A serviceman with a gunshot wound of the upper third of the right leg with damage to the main vessels was brought to the medical company. Systolic blood pressure - 100 mm Hg, Ps - 90/min. When providing first aid, a tourniquet and a bandage were applied to the wound an hour ago, and an anesthetic agent was administered. In which order should the injured be evacuated to the next stage of medical evacuation?

Підлягає амбулаторному лікуванню Subject to outpatient treatment

Не підлягає евакуації Not subject to evacuation

В першу чергу First of all

В другу чергу Secondly

Залишається для лікування на даному етапі медичної евакуації Remains for treatment at this stage of medical evacuation